Exam Simulation - Test 1 Flashcards

1
Q

An assumption underlying Marlatt and Gordon’s (1985) relapse prevention (RP) model is that:
Select one:

A.
abstinence is the only goal since any drug use will trigger the “latent disease.”

B.
addictive behaviors are acquired through classical conditioning and, therefore, must be eliminated through the use of aversive conditioning and other classical conditioning techniques.

C.
addictive habit patterns can be changed through the application of self-management and self-control procedures.

D.
relapse is a result of a “treatment failure” rather than an “individual failure.”

A

Answer C is correct: Marlatt and Gordon’s relapse prevention model is based on social learning theory and views addictive behaviors as acquired (overlearned) habit patterns. The focus of their approach to relapse prevention is on acquiring the skills needed to reduce the risk for relapse.

Answer A: This is more consistent with the disease model of addiction.

Answer B: Although Marlatt and Gordon’s model recognizes the role of classical conditioning in the acquisition and maintenance of addictive behaviors, aversive conditioning is not a key relapse prevention technique.

Answer D: Relapse is not considered either a treatment or an individual failure but as a “transitional process” that may or may not be followed by a return to baseline levels of the target behavior.
The correct answer is: addictive habit patterns can be changed through the application of self-management and self-control procedures.

How well did you know this?
1
Not at all
2
3
4
5
Perfectly
2
Q

The magnitude of the standard error of means decreases when:
Select one:

A.
the sample size increases and the population standard deviation decreases.

B.
the sample size decreases and the population standard deviation increases.

C.
the number of samples increases and the population standard deviation decreases.

D.
the number of samples decreases and the population standard deviation increases.

A

Knowing the formula for the standard error of means would have helped you identify the correct answer to this question: The standard error of means is calculated by dividing the standard deviation of the population by the square root of the sample size.

a. CORRECT Decreasing the numerator (population standard deviation) and/or increasing the sample size would reduce the size of the standard error of means.
b. Incorrect This would increase the size of the standard error of means.
c. Incorrect It is the size of the sample, not the number of samples, that affects the magnitude of the standard error of means.
d. Incorrect See explanation above.

The correct answer is: the sample size increases and the population standard deviation decreases.

How well did you know this?
1
Not at all
2
3
4
5
Perfectly
3
Q

Older adults often experience an advanced sleep phase which means that they:
Select one:

A.
begin a sleep period with REM (versus non-REM) sleep.

B.
begin a sleep period with Stage 3 sleep.

C.
have trouble falling asleep at night and wake up later in the morning.

D.
get sleepier earlier in the evening and wake up earlier in the morning.

A

As noted in the Physiological Psychology and Psychopharmacology chapter of the written study materials, sleep patterns vary with age. For the exam, you want to be familiar with the patterns characteristic of infants and older adults.

a. Incorrect See explanation for response d.
b. Incorrect See explanation for response d.
c. Incorrect See explanation for response d.
d. CORRECT Older adults experience an advanced sleep phase, which refers to a shift in the timing of sleep that involves getting sleepier earlier in the evening and waking up earlier in the morning.

The correct answer is: get sleepier earlier in the evening and wake up earlier in the morning.

How well did you know this?
1
Not at all
2
3
4
5
Perfectly
4
Q

Of the intermittent schedules of reinforcement, the variable ratio schedule produces the quickest acquisition of a behavior and the greatest resistance to extinction once reinforcement is terminated. This is because reinforcement is presented:
Select one:

A.
after a predictable number of responses.

B.
after an unpredictable number of responses.

C.
after a predictable interval of time.

D.
after an unpredictable interval of time.

A

Ratio schedules provide reinforcement after a prespecified number of responses, while interval schedules provide reinforcement after a prespecified period of time (as long as at least one response is made during that interval). Knowing this would have helped you choose the correct answer to this question.

a. Incorrect This describes a fixed ratio schedule.
b. CORRECT When using a variable ratio schedule, the number of responses required for reinforcement varies from trial to trial but, on the average, requires a specific number of responses. For example on a VR-10 schedule, the individual might be reinforced for 8 responses, then 12 responses, and then 10 responses. It is apparently this unpredictability that accounts for the effectiveness of this schedule in establishing and maintaining behaviors.
c. Incorrect This describes a fixed interval schedule.
d. Incorrect This describes a variable interval schedule.

The correct answer is: after an unpredictable number of responses.

How well did you know this?
1
Not at all
2
3
4
5
Perfectly
5
Q

A meta-analysis of the research conducted by Roberts, Walton, and Viechtbauer (2006) found that scores on which of the following personality traits continue to increase into later adulthood?
Select one:

A.
agreeableness and conscientiousness

B.
neuroticism and social vitality

C.
social vitality and agreeableness

D.
openness to experience and conscientiousness

A

The results of Roberts, Walton, and Viechtbauer’s (2006) meta-analysis is summarized in the Lifespan Development chapter of the written study materials.

a. CORRECT These investigators found that agreeableness, conscientiousness, social dominance, and emotional stability continue to increase over the lifespan.
b. Incorrect See explanation above.

c. Incorrect See explanation above.
d. Incorrect See explanation above.

The correct answer is: agreeableness and conscientiousness

How well did you know this?
1
Not at all
2
3
4
5
Perfectly
6
Q

Obsessive-Compulsive Disorder is more common in males than in females among those aged:
Select one:

A.
8 to 12.

B.
22 to 26.

C.
38 to 42.

D.
52 to 56.

A

Answer A is correct: The rates of OCD for males and females are about equal among adolescents and adults. However, because the onset of the disorder is earlier for males than for females, OCD is more prevalent among male children than female children.

Answers B, C, and D: See explanation for answer A.

The correct answer is: 8 to 12.

How well did you know this?
1
Not at all
2
3
4
5
Perfectly
7
Q

Children ordinarily first show signs that they have lost their ability to distinguish between speech sounds that are not common in their native language by _________ months of age.
Select one:

A.
1 to 2

B.
5 to 6

C.
9 to 11

D.
14 to 16

A

The research has found that infants distinguish between speech sounds at a very early age, including sounds that are not part of their native language.

a. Incorrect See explanation for response c.
b. Incorrect See explanation for response c.
c. CORRECT Babies can initially distinguish between speech sounds that are and are not part of their own language. This ability is subsequently lost, however, around the same time they begin to understand meaningful speech - i.e., at about 9 to 11 months of age. See, e.g., D. E. Papalia and S. W. Olds, Human development, New York: McGraw-Hill, 1995.
d. Incorrect See explanation for response c.

The correct answer is: 9 to 11

How well did you know this?
1
Not at all
2
3
4
5
Perfectly
8
Q

Research investigating the relationship between age and crystallized and fluid intelligence has found that:
Select one:

A.
crystallized and fluid intelligence both increase until the late thirties to early forties but thereafter decline.

B.
crystallized intelligence begins to decline in the late twenties to early thirties, while fluid intelligence does not begin to decline until the late fifties or early sixties.

C.
fluid intelligence begins to decline in the late twenties to early thirties, while crystallized intelligence continues to increase until age 60 or later.

D.
fluid and crystallized intelligence remain relatively stable until the late forties but crystallized intelligence begins to decline by the mid to late fifties.

A

A number of studies have confirmed age-related trends in crystallized and fluid intelligence. Answer C is correct. The exact timing of age-related changes in fluid and crystallized intelligence varies somewhat from study to study but, in general, the studies have found that fluid intelligence begins to decline at a relatively early age, while crystallized intelligence continues to increase into late adulthood. This answer best describes the results of the research.

The correct answer is: fluid intelligence begins to decline in the late twenties to early thirties, while crystallized intelligence continues to increase until age 60 or later.

How well did you know this?
1
Not at all
2
3
4
5
Perfectly
9
Q

The final group product is most affected by the performance of the member who makes the smallest contribution on which type of task?
Select one:

A.
compensatory

B.
disjunctive

C.
conjunctive

D.
additive

A

As noted in the Industrial-Organizational Psychology chapter of the written study materials, group tasks can be describd as compensatory, disjunctive, conjunctive, or additive.

a. Incorrect See explanation for response c.
b. Incorrect See explanation for response c.
c. CORRECT On a conjunctive task, the group product is limited by the contribution of the worst-performing member.
d. Incorrect See explanation for response c.
d. Incorrect See explanation for response c.

The correct answer is: conjunctive

How well did you know this?
1
Not at all
2
3
4
5
Perfectly
10
Q

“Criterion deficiency” refers to:
Select one:

A.
the degree to which the conceptual criterion is not measured by the actual criterion.

B.
the degree to which the actual criterion systematically measures something other than the conceptual criterion.

C.
the degree to which the actual criterion is entirely unrelated to the conceptual criterion.

D.
the degree to which the actual criterion provides inconsistent information about the conceptual criterion.

A

For the exam, you want to be familiar with several terms that are used to describe the adequacy of a criterion measure. These are described in the Industrial-Organizational Psychology chapter of the written study materials.

a. CORRECT This is the correct definition of criterion deficiency. An actual criterion (the criterion measure) is deficient to the extent that it does not measure the conceptual (or hypothetical) criterion. In other words, job performance is due to several factors, but most criterion measures are deficient because they measure only one or two of those factors.
b. Incorrect This response describes one aspect of criterion contamination.
c. Incorrect This describes another aspect of criterion contamination.
d. Incorrect This describes the unreliability of a criterion measure.

The correct answer is: the degree to which the conceptual criterion is not measured by the actual criterion.

How well did you know this?
1
Not at all
2
3
4
5
Perfectly
11
Q

The analysis of covariance (ANCOVA) is used to:
Select one:

A.
analyze data when a “fully crossed” research design has been used.

B.
analyze data when a study involves two or more independent variables.

C.
statistically remove the effects of an extraneous variable.

D.
statistically analyze the main and interaction effects of an extraneous variable.

A

The ANCOVA is one of several forms of the analysis of variance that you want to be familiar with for the licensing exam.

a. Incorrect See explanation for response c.
b. Incorrect See explanation for response c.
c. CORRECT The ANCOVA is used to statistically remove the effects of an extraneous variable on the dependent variable so that the effects of one or more independent variables can be more easily detected.
d. Incorrect The MANOVA is used to statistically analyze the main and interaction effects of an extraneous variable.

The correct answer is: statistically remove the effects of an extraneous variable.

How well did you know this?
1
Not at all
2
3
4
5
Perfectly
12
Q

Which of the following is a cause of conductive deafness?
Select one:

A.
infection of the middle ear

B.
damage to the hair cells

C.
cochlear damage

D.
lesions in the auditory cortex

A

There are three types of deafness: conductive, sensorineural, and central.

a. CORRECT Conductive deafness results from failure of mechanical stimulation to reach the cochlea and can be caused by an infection or obstruction in the outer or middle ear.
b. Incorrect This refers to sensorineural deafness.
c. Incorrect Damage to the cochlea results in sensorineural deafness.
d. Incorrect This is a cause of central deafness.

The correct answer is: infection of the middle ear

How well did you know this?
1
Not at all
2
3
4
5
Perfectly
13
Q

Which of the following aspects of the central nervous system is least well-developed at birth?
Select one:

A.
diencephalon

B.
midbrain

C.
cerebral cortex

D.
cerebellum

A

The brain develops both before and after birth in an orderly sequence. At birth, the infant’s brain is about one-fourth the size of an adult brain.

a. Incorrect The diencephalon consists of the thalamus and hypothalamus and is sufficiently well-developed at birth to allow for critical functions that are necessary for life.
b. Incorrect Like the diencephalon, the midbrain is relatively well-developed at birth.
c. CORRECT The cortex is not well-developed at birth, which suggests that newborn behaviors are primarily reflexive (i.e., mediated by the lower centers of the brain). Some areas of the cortex (e.g., those involved in problem-solving, self-concept, and planning) do not fully develop until adolescence or early adulthood.
d. Incorrect The cerebellum is involved in motor coordination, and the motor areas of the brain develop prior to the areas governing the higher cognitive functions.

The correct answer is: cerebral cortex

How well did you know this?
1
Not at all
2
3
4
5
Perfectly
14
Q

According to Wolfgang Kohler, learning:
Select one:

A.
is largely the result of trial-and-error.

B.
is the result of repeated exposure to the same or similar conditions.

C.
involves using effective encoding and retrieval cues.

D.
involves discovering relationships between elements of the problem.

A

Kohler is associated with insight learning, which is a sudden understanding of relationships among a set of elements relevant to a problem that leads to recognition of the solution to the problem.

a. Incorrect Kohler did not assume that learning is the result of trial-and-error.
b. Incorrect This isn’t compatible with Kohler’s theory.
c. Incorrect This doesn’t describe insight learning.
d. CORRECT Insight learning occurs when there are relationships in the problem to be discovered and the discovery of those relationships is within the organism’s cognitive capacity.

The correct answer is: involves discovering relationships between elements of the problem.

How well did you know this?
1
Not at all
2
3
4
5
Perfectly
15
Q

Predictors of a positive response to a tricyclic antidepressant as a treatment for Major Depressive Disorder include all of the following except:
Select one:

A.
insidious onset of symptoms.

B.
weight loss.

C.
middle and late insomnia.

D.
delusions.

A

The various antidepressants, to some degree, are most effective for different depressive symptoms.

a. Incorrect See explanation for response d.
b. Incorrect See explanation for response d.
c. Incorrect See explanation for response d.
d. CORRECT Hysterical traits, multiple previous episodes, and delusions are associated with a poor response to the tricyclic drugs. The symptoms listed in responses a, b, and c (along with psychomotor disturbances and other vegetative symptoms) are predictive of a positive response.

The correct answer is: delusions.

How well did you know this?
1
Not at all
2
3
4
5
Perfectly
16
Q

Which of the following sleep abnormalities have been linked to Major Depressive Disorder?
Select one:

A.
increased Stage 3 and Stage 4 sleep

B.
decreased duration of REM sleep early in the night

C.
early onset of REM sleep

D.
shortened sleep latency

A

Answer C is correct. A decreased REM latency (earlier onset of REM sleep) is one of several sleep abnormalities associated with MDD.

Answers A, B, and D: The abnormalities listed in these answers are the opposite of what is true – i.e., MDD is associated with decreased Stage 3 and Stage 4 sleep, an increased duration of REM sleep early in the night, and a prolonged sleep latency.

The correct answer is: early onset of REM sleep

How well did you know this?
1
Not at all
2
3
4
5
Perfectly
17
Q

Research investigating comorbidity in children suggests that, when depression occurs in conjunction with ____________, the depression is often associated with a different course and a different family background than when it occurs alone.
Select one:

A.
an Anxiety Disorder

B.
Conduct Disorder

C.
somatic complaints

D.
learning problems

A

Answer B is correct: Some experts suggest that depression occurring in conjunction with Conduct Disorder is actually a different type of depression since it is associated with a lower rate of depression in adulthood as well as with a lower rate of depression among relatives. See R. Harrington, Affective disorders, in M. Rutter, et al., (Eds.), Child and Adolescent Psychiatry,Oxford, Blackwell Scientific Publications, 1994. Note that this is a difficult “distant galaxy” question that addresses a topic that few people are familiar with. For these kinds of questions, if you don’t know the answer, don’t spend too much time trying to figure them out: Make an “educated guess” and move on to the next question.

Answer A: Anxiety commonly occurs in conjunction with depressive symptoms in children. The research suggests that when anxiety and depression occur together, the depression is similar in terms of course and family background to depression that occurs without anxiety.

Answer C: There is currently no research suggesting that somatic complaints are indicative of a different form of depression.

Answer D: There is currently no evidence that suggests that learning problems are indicative of a different form of depression.

The correct answer is: Conduct Disorder

How well did you know this?
1
Not at all
2
3
4
5
Perfectly
18
Q

A test developer would be interested in the selection ratio, base rate, and validity coefficient of a test when she is evaluating the test’s:
Select one:

A.
differential validity.

B.
external validity.

C.
incremental validity.

D.
concurrent validity.

A

The selection ratio, base rate, and validity coefficient are used to estimate a test’s incremental validity using the Taylor-Russell tables.

a. Incorrect See explanation for response c.
b. Incorrect See explanation for response c.
c. CORRECT Information on these three factors would allow the researcher to estimate a test’s incremental validity (i.e., the degree to which use of the test will increase decision-making accuracy).
d. Incorrect See explanation for response c.

The correct answer is: incremental validity.

How well did you know this?
1
Not at all
2
3
4
5
Perfectly
19
Q

According to George Kelly (1995), a fully functioning person is one:
Select one:

A.
whose predictions about events and other people are usually correct.

B.
whose self and self-image are in a state of congruence.

C.
who is able to first identify and then achieve realistic goals in a responsible manner.

D.
who routinely experiences events with here-and-now immediacy.

A

Knowing that Kelly’s personal construct theory focuses on how a person “construes” (perceives, interprets, and predicts) events would have helped you identify the correct answer to this question.

a. CORRECT The primary goals of personal construct therapy are to identify and then revise the client’s maladaptive personal constructs so that he/she makes better predictions about events and other people.
b. Incorrect See explanation for response a.
c. Incorrect See explanation for response a.
d. Incorrect See explanation for response a.

The correct answer is: whose predictions about events and other people are usually correct.

How well did you know this?
1
Not at all
2
3
4
5
Perfectly
20
Q

Research by Patterson and his colleagues (1992) at the Oregon Social Learning Center has linked high levels of aggression in children to which of the following?
Select one:

A.
lack of empathy

B.
sociocultural norms related to aggression

C.
coercive family interactions

D.
emotional over-reactivity

A

Patterson and colleagues (1992) focus on family contributions to aggression.

a. Incorrect See explanation for response c.
b. Incorrect See explanation for response c.
c. CORRECT These investigators have concluded that families of highly aggressive children are characterized by a high degree of coercive family interactions and poor parental monitoring of children’s activities.
d. Incorrect See explanation for response c.

The correct answer is: coercive family interactions

How well did you know this?
1
Not at all
2
3
4
5
Perfectly
21
Q

The use of clomipramine in the treatment of Obsessive-Compulsive Disorder (OCD):
Select one:

A.
is effective only for reducing accompanying depressive symptoms.

B.
has anti-obsessional effects both during administration and after cessation of drug use.

C.
has both antidepressant and anti-obsessional effects for as long as the drug is administered.

D.
eliminates the need for psychosocial treatment, especially in mild to moderate cases of OCD.

A

Answer C is correct: Research has shown that some antidepressants - including the tricyclic clomipramine - not only reduce the depressive symptoms that often accompany OCD but also reduce obsessions. However, when clomipramine is not used in conjunction with exposure with response prevention, symptoms typically return soon after the drug is stopped.

Answers A, B, and D: See explanation for answer C.

The correct answer is: has both antidepressant and anti-obsessional effects for as long as the drug is administered.

How well did you know this?
1
Not at all
2
3
4
5
Perfectly
22
Q

Erikson identified identity formation as the key issue during adolescence. At around age 18 or 19, this issue begins to be replaced by which of the following?
Select one:

A.
the need to “repudiate one’s choices.”

B.
the need to develop interpersonal closeness and solidarity.

C.
the need to achieve a higher degree of individuation.

D.
the need to be productive and creative.

A

Erikson’s stage of identity vs. role confusion is followed by intimacy vs. isolation. See the Lifespan Development chapter of the written study materials for additional information on Erikson’s stages of psychosocial development.

a. Incorrect Repudiation is part of identity formation.
b. CORRECT The psychosocial issues of early adulthood center on intimacy and solidarity versus isolation.
c. Incorrect Individuation is a concept associated with Levinson, not Erikson.
d. Incorrect The stage of generativity vs. stagnation is characteristic of middle adulthood.

The correct answer is: the need to develop interpersonal closeness and solidarity.

How well did you know this?
1
Not at all
2
3
4
5
Perfectly
23
Q

Older workers are LEAST likely to show decrements on which of the following types of work-related tasks as the result of normal age-related cognitive decline?
Select one:

A.
verbal tasks

B.
dual (multi) tasks

C.
tasks requiring inductive reasoning

D.
tasks requiring selective attention

A

Research has confirmed that certain aspects of cognition are more susceptible to age-related cognitive decline than other aspects are. See, e.g., A. Laville, Elderly workers, in Encyclopaedia of occupational health and safety (4th ed.), Washington, International Labour Office, 1998.

a. CORRECT Verbal ability has been found to be relatively immune to the effects of normal aging and does not begin to show declines until the late 70s or early 80s.
b. Incorrect The research has generally confirmed that increasing age is associated with decreased effectiveness on dual (multi) tasks, most likely because performance on these tasks depends on working memory, divided attention, and other abilities that are adversely affected by age-related cognitive decline.
c. Incorrect Inductive reasoning depends on fluid intelligence, which is affected by normal aging.
d. Incorrect Selective attention is also adversely affected by increasing age.

The correct answer is: verbal tasks

How well did you know this?
1
Not at all
2
3
4
5
Perfectly
24
Q

Men with paraplegia as the result of a spinal cord injury:
Select one:

A.
can achieve an erection but cannot ejaculate or experience orgasm.

B.
can achieve an erection and ejaculate but cannot experience orgasm.

C.
can achieve an erection and may be able to ejaculate and experience orgasm.

D.
cannot achieve an erection, ejaculate, or experience orgasm.

A

This is a difficult question because the consequences of spinal cord injury vary from individual to individual.

a. Incorrect See explanation for response c.
b. Incorrect See explanation for response c.
c. CORRECT Of the answers given, this one provides the best generalization. Even with high spinal cord lesions, many men are able to experience reflexive erections; and with low spinal legions, many men experience psychic erections and ejaculation. In addition, a substantial proportion report having orgasms or sensations similar to orgasm.
d. Incorrect See explanation for response c.

The correct answer is: can achieve an erection and may be able to ejaculate and experience orgasm.

How well did you know this?
1
Not at all
2
3
4
5
Perfectly
25
Q

Which of the following occupational themes is most dissimilar to Holland’s social theme?
Select one:

A.
enterprising

B.
realistic

C.
conventional

D.
investigative

A

Knowing that the occupational themes are listed in order of similarity in Holland’s hexagon and that the order is represented by the acronym “RIASEC” would have helped you identify the correct answer to this question.

a. Incorrect See explanation for response b.
b. CORRECT The social theme is directly opposite the realistic theme in Holland’s hexagon, which indicates that the realistic theme is most unlike the social theme.
c. Incorrect See explanation for response b.
d. Incorrect See explanation for response b.

The correct answer is: realistic

How well did you know this?
1
Not at all
2
3
4
5
Perfectly
26
Q

In an experiment, a tone is presented to a dog just before meat powder is presented numerous times so that, eventually, the dog salivates when the tone is presented alone. Then a light and the tone are simultaneously presented just before the meat powder numerous times. Based on your knowledge of “blocking,” you predict that the dog will subsequently:
Select one:

A.
salivate in response to the light when it is presented alone and continue to salivate in response to the tone when it is presented alone.

B.
salivate in response to the light when it is presented alone but salivate in response to the tone only when it is presented with the light.

C.
continue to salivate in response to the tone but not salivate in response to the light.

D.
not salivate in response to either the tone or the light.

A

Blocking occurs when one stimulus blocks the other from becoming a conditioned stimulus.

a. Incorrect See explanation for response c.
b. Incorrect See explanation for response c.
c. CORRECT In this situation, the tone will block the light from becoming a CS since the tone was the first stimulus to be paired with the meat powder. Apparently, blocking occurs because the second stimulus (the light in this case) provides redundant information and does not become linked to the unconditioned stimulus (meat powder).
d. Incorrect See explanation for response c.

The correct answer is: continue to salivate in response to the tone but not salivate in response to the light.

How well did you know this?
1
Not at all
2
3
4
5
Perfectly
27
Q

According to Hersey and Blanchard’s situational leadership model, a manager will be most effective with an employee who is low in both ability and motivation when the manager:
Select one:

A.
provides specific instructions and closely supervises the employee’s work.

B.
encourages the employee to take responsibility for decision-making.

C.
shares ideas with the employee and supports the employee’s participation in decision-making.

D.
acts as a supportive coach.

A

Situational leadership theory proposes that, to be effective, managers must adopt a leadership style that matches the employee’s maturity, which is determined by a combination of the employee’s ability and motivation.

a. CORRECT This describes a “telling” style, which, according to Hersey and Blanchard, is appropriate for employees who are low in both ability and motivation.
b. Incorrect This describes a “delegating” style, which is best for employees who are high in both ability and motivation.
c. Incorrect This is characteristic of the “participating” style, which is best for employees who are high in ability but low in motivation.
d. Incorrect This is the “selling” style, which works best for employees who are low in ability but high in motivation.

The correct answer is: provides specific instructions and closely supervises the employee’s work.

How well did you know this?
1
Not at all
2
3
4
5
Perfectly
28
Q

For practitioners of humanistic psychotherapy, psychopathology is the result of:
Select one:

A.
blocked potential.

B.
dis-integration.

C.
unresolved conflicts.

D.
severe trauma.

A

Therapists classified as humanists share a belief in the inherent capacity for humans to grow toward the achievement of their potential (i.e., to achieve self-actualization).

a. CORRECT Neurosis and psychosis are generally viewed as the result of interference with the natural potential for growth and self-actualization.
b. Incorrect See explanation for response a.
c. Incorrect See explanation for response a.
d. Incorrect See explanation for response a.

The correct answer is: blocked potential.

How well did you know this?
1
Not at all
2
3
4
5
Perfectly
29
Q

Which of the following is an example of anterograde amnesia?
Select one:

A.
As the result of a head injury he received in a car accident, a man cannot remember where he was going before the accident occurred.

B.
A woman cannot remember how she got to the hospital or other events that occurred during the 24 hours after she was sexually assaulted.

C.
A high school junior who learned Spanish her freshman year is having trouble learning French because she keeps substituting Spanish words for French ones.

D.
A college student can remember information related to an important event but cannot remember how he acquired that information.

A

Anterograde amnesia involves a loss of memory for events that occur after the event that caused the memory loss.

a. Incorrect A loss of memory for events that occurred prior to the trauma that caused the memory impairment is referred to as retrograde amnesia.
b. CORRECT This answer provides an example of anterograde amnesia - i.e., the woman has no memory for events that occurred after the assault.
c. Incorrect This is an example of proactive interference.
d. Incorrect This is an example of source amnesia.

The correct answer is: A woman cannot remember how she got to the hospital or other events that occurred during the 24 hours after she was sexually assaulted.

How well did you know this?
1
Not at all
2
3
4
5
Perfectly
30
Q

Studies investigating the relationship between maternal socialization practices and children’s internalization of conscience have found that:
Select one:

A.
maternal gentle discipline is predictive of internalization regardless of a child’s temperament.

B.
maternal gentle discipline is predictive of internalization only for children who have a secure attachment.

C.
maternal gentle discipline is predictive of internalization for children who are fearful and anxious prone.

D.
maternal gentle discipline is predictive of internalization for children who are relatively fearless and non-anxious.

A

This question is asking about research by G. Kochanska, which found that the relationship between maternal parenting style and children’s internalization of conscience is mediated by the child’s temperament. (Socialization and temperament in the development of guilt and conscience, Child Development, 62, 1379-1392, 1991).

a. Incorrect See explanation for response c.
b. Incorrect See explanation for response c.
c. CORRECT The impact of gentle discipline (use of control that de-emphasizes power assertion and emphasizes reasoning, suggestion, distraction, and positive reinforcement) is most effective in terms of internalization for children who are relatively fearful and anxious.
d. Incorrect For children who are fearless and non-anxious, a secure attachment seems to be more important than maternal parenting style for internalization of conscience.

The correct answer is: maternal gentle discipline is predictive of internalization for children who are fearful and anxious prone.

How well did you know this?
1
Not at all
2
3
4
5
Perfectly
31
Q

The goodness-of-fit model proposed by Thomas and Chess (1977) is supported by which of the following research findings?
Select one:

A.
Modeling and imitation are primary contributors to the development of a gender role identity.

B.
Providing a child with experiences that are slightly beyond his/her abilities maximizes opportunities for social and cognitive development.

C.
Parents who adopt an authoritative style are likely to have children who obtain high scores on measures of self-esteem, peer popularity, and scholastic achievement.

D.
Irritable temperament in children is predictive of insecure attachment only when mothers are not socially supportive.

A

Thomas and Chess proposed that childhood pathology is related to a lack of fit between the child’s basic temperament and the parents’ behaviors toward the child.

a. Incorrect See explanation for response d.
b. Incorrect See explanation for response d.
c. Incorrect See explanation for response d.
d. CORRECT This finding is consistent with the prediction made by Thomas and Chess - i.e., it is the combination of the child’s temperament and the parents’ parenting style that determines the child’s developmental outcomes.

The correct answer is: Irritable temperament in children is predictive of insecure attachment only when mothers are not socially supportive.

How well did you know this?
1
Not at all
2
3
4
5
Perfectly
32
Q

Damage to the right hippocampus is most likely to cause:
Select one:

A.
psychic blindness.

B.
interruption of the normal sleep-wake cycle.

C.
deficits in spatial memory.

D.
deficits in procedural memory.

A

The hippocampus is vital to learning and memory.

a. Incorrect Psychic blindness is a symptom of Kluver-Bucy syndrome which is caused by damage to the amygdala and temporal lobes.
b. Incorrect The research has not linked the hippocampus to the sleep-wake cycle.
c. CORRECT The right hippocampus is involved in memory for nonverbal information, including spatial and temporal memory. For example, in a study of cabdrivers, PET scans revealed that the right hippocampus was responsible for the drivers’ ability to form and recall complex routes.
d. Incorrect The hippocampus plays a role in declarative memory but does not appear to be responsible for the formation of procedural memories.

The correct answer is: deficits in spatial memory.

How well did you know this?
1
Not at all
2
3
4
5
Perfectly
33
Q

Which of the following best describes the course of Borderline Personality Disorder (BPD)?
Select one:

A.
Symptoms are most severe in early adulthood and, for many individuals, decrease substantially in severity by middle-age.

B.
Symptoms continue to increase in severity from early to middle-adulthood but thereafter remain stable for most individuals.

C.
Symptoms wax and wane throughout the lifespan with the most severe symptoms occuring during middle-age.

D.
Symptoms remain relatively stable throughout the lifespan with some waxing and waning in severity of symptoms that is related to factors in the individual’s environment.

A

Answer A is correct: A common pattern is for symptoms to be most severe and chronic in early adulthood with substantial improvement in functionng during the 30s and 40s and with most individuals no longer meeting the full diagnostic criteria for BPD by age 40.

Answers B, C, and D: See explanation for answer A.
The correct answer is: Symptoms are most severe in early adulthood and, for many individuals, decrease substantially in severity by middle-age.

How well did you know this?
1
Not at all
2
3
4
5
Perfectly
34
Q

When a test’s reliability coefficient is equal to ____, this means that 81% of variability in test scores is attributable to true score variability.
Select one:

A.
0.9

B.
0.81

C.
0.65

D.
0.19

A

Knowing that a reliability coefficient is interepreted directly in terms of true score variability would have enabled you to identify the correct answer to this question.

a. Incorrect See explanation for response b.
b. CORRECT When the reliability coefficient is equal to .81, this means that 81% of variability in test scores reflects true score variability and 10% reflects measurement error.
c. Incorrect See explanation for response b.
d. Incorrect See explanation for response b.

The correct answer is: 0.81

How well did you know this?
1
Not at all
2
3
4
5
Perfectly
35
Q

The vasovagel response associated with Specific Phobia, blood-injection-injury type is characterized by:
Select one:

A.
a drop in heart rate and blood pressure with xerosis and ruddiness.

B.
a sustained increase in heart rate and blood pressure with hypervigilence and xerosis.

C.
a sustained increase in heart rate and blood pressure with ruddiness and tinnitus.

D.
a drop in heart rate and blood pressure with sweating, pallor, and fainting.

A

Answer D is correct: The blood-injection-injury type of Specific Phobia produces a response to feared stimuli that differs from the response associated with other types of this disorder. The vasovagel response associated with this type involves a brief initial increase in heart rate and blood pressure followed by a decrease in heart rate and blood pressure that may lead to fainting. Other characteristic symptoms include sweating, pallor, nausea, and tinnitus.

Answer A: Although the vasovagal response does involve a drop in heart rate and blood pressure, it is not characterized by xerosis (dry skin) or ruddiness (blushing or redness of the skin).

Answer B: The increase in heart rate and blood pressure associated with this type of Specific Phobia is temporary and brief, not sustained.

Answer C: See explanation for answer D.

The correct answer is: a drop in heart rate and blood pressure with sweating, pallor, and fainting.

How well did you know this?
1
Not at all
2
3
4
5
Perfectly
36
Q

Stefan S., age 42, was laid off from his job seven months ago and has been unable to find a new one. Although Stefan is receiving some financial help from his widowed mother, he lost his new car because he wasn’t able to make the payments and is living in a studio apartment in his sister’s basement. His sister’s husband is very unhappy with this arrangement and is constantly calling Stefan a “worthless bum.” In terms of Maslow’s need hierarchy theory, Stefan’s most prepotent needs are related to:
Select one:

A.
safety/security.

B.
esteem/status.

C.
belonging.

D.
existence.

A

Maslow’s need hierarchy theory identifies five basic human needs: physiological, safety, belonging, status, and self-actualization. It proposes that, in terms of motivation, lower-order needs take precedence over higher-order needs.

a. CORRECT Stefan’s physiological needs (needs related to physical survival) are being met, but his safety needs are not - i.e., he does not have a job and is living in a somewhat “hostile” environment. Therefore, according to Maslow’s theory, Stefan’s safety/security needs are the primary motivators of his behavior.
b. Incorrect According to Maslow’s theory, esteem/status needs do not act as motivators until belonging needs have been satisfied.
c. Incorrect Belonging needs do not become motivators until safety needs have been satisfied.
d. Incorrect Alderfer distinguishes between three basic human needs - existence, relatedness, and growth. Alderfer’s theory predicts that existence needs would be motivators for Stefan, but this question is asking about Maslow’s theory, so answer a is the best response.

The correct answer is: safety/security.

How well did you know this?
1
Not at all
2
3
4
5
Perfectly
37
Q

Research investigating compliance behaviors of individuals with diabetes mellitus indicates that adherence to dietary and other treatment regimens is most problematic:
Select one:

A.
during the elementary school years.

B.
during preadolescence.

C.
during adolescence.

D.
among older adults.

A

Answer C is correct: Research on diabetes mellitus and other chronic illnesses has consistently shown that compliance is most problematic during adolescence. Although adolescents are more knowledgeable about their illness than younger children are, they typically exhibit a lower level of compliance.

Answers A, B, and D: See explanation for answer C.

The correct answer is: during adolescence.

How well did you know this?
1
Not at all
2
3
4
5
Perfectly
38
Q

As used by Lorenz (1965), the term “critical period” refers to:
Select one:

A.
the period from ages three to six months during which the basic structure of the personality is formed.

B.
periods when the infant shows predictable “growth spurts.”

C.
the third trimester when the brain is at greatest vulnerability to structural damage.

D.
a period shortly after birth during which an infant ordinarily bonds with its mother.

A

The term “critical period” refers to a period when a specific experience will have its greatest impact on development. Lorenz applied the term to the period during which geese bond with (imprint on) their mothers.

a. Incorrect See explanation for response c.
b. Incorrect See explanation for response c.
c. Incorrect See explanation for response c.
d. CORRECT Lorenz found that the “critical period” for imprinting in geese is 2-3 days after birth. In other words, geese will stay close (attach) to stimuli that they are exposed to two to three days after birth. Research investigating the existence of a similar critical period in humans has been inconclusive.

The correct answer is: a period shortly after birth during which an infant ordinarily bonds with its mother.

How well did you know this?
1
Not at all
2
3
4
5
Perfectly
39
Q

A middle-aged man frequently experiences vivid and occasionally frightening hallucinations just as he’s beginning to fall asleep. This condition is best described as:
Select one:

A.
sleep terrors.

B.
sleep apnea.

C.
hypnagogic hallucinations.

D.
hypnopompic hallucinations.

A

Answer C is correct: Hypnagogic hallucinations may occur in Narcolepsy, and they are vivid hallucinations that occur during the transition from wakefulness to sleep.

Answers A and B: See explanation for answer C.

Answer D: Hypnopompic hallucinations occur during the transition from sleep to wakefulness. These may also be present in Narcolepsy.

The correct answer is: hypnagogic hallucinations.

How well did you know this?
1
Not at all
2
3
4
5
Perfectly
40
Q

The Health Belief Model predicts that, to modify an individual’s health-related behaviors, an intervention should focus on:
Select one:

A.
the external rewards associated with being healthy.

B.
the severity, duration, and stage of the individual’s illness.

C.
the individuals locus of control.

D.
the individuals knowledge, motivation to act, and self-efficacy beliefs.

A

As its name implies, the Health Belief Model focuses on the impact of an individual’s beliefs on his/her health-related behaviors.

a. Incorrect The Health Belief Model emphasizes the individual’s perceptions and beliefs rather than external conditions (e.g., rewards and punishments) that influence health-related behaviors.
b. Incorrect This is not a focus of interventions based on the Health Belief Model, which usually emphasize prevention of a disorder rather than the treatment of an existing disorder.
c. Incorrect The Health Locus of Control Model emphasizes the role of locus of control beliefs.
d. CORRECT According to the Health Belief Model, a person’s willingness to take appropriate health-related actions is related to the person’s beliefs about (1) his/her susceptibility to the illness; (2) the consequences of the illness; and (3) the benefits of and barriers to taking appropriate action. Interventions based on the Health Belief Model focus on the individual’s knowledge about the illness and methods for avoiding it; motivation to take action; and self-efficacy beliefs.

The correct answer is: the individuals knowledge, motivation to act, and self-efficacy beliefs.

How well did you know this?
1
Not at all
2
3
4
5
Perfectly
41
Q

In the context of exposure to the human immunodeficiency virus (HIV), the term “seroconversion” refers to:
Select one:

A.
the point in time at which transmission of the virus occurred.

B.
the process of converting from antibody negative to antibody positive.

C.
the period during which the infected individual is asymptomatic.

D.
the bodys reaction to an anti-retroviral drug.

A

As a general term, seroconversion refers to the development of antibodies to a particular antigen (e.g., HIV).

a. Incorrect See explanation for response b.
b. CORRECT In the context of HIV, seroconversion refers to the conversion from HIV negative (seronegative) to HIV positive (seropositive) as the result of the presence of antibodies. Note that, because seroconversion often does not occur until several weeks after infection, an infected individual may develop symptoms of acute HIV infection while still having a negative HIV antibody test.
c. Incorrect See explanation for response b.
d. Incorrect See explanation for response b.

The correct answer is: the process of converting from antibody negative to antibody positive.

How well did you know this?
1
Not at all
2
3
4
5
Perfectly
42
Q

An 11-year-old loves playing the piano and does so for at least one hour a day. His parents have begun to reward him for doing so by giving him a dollar for each 30 minutes that he practices and find that this increases the amount of time he spends playing the piano each day. Subsequently, the child’s parents decide to discontinue the reward. Research by Lepper, Greene, and Nisbett (1973) suggests that, once the parents discontinue the reward, which of the following will occur:
Select one:

A.
the child will continue playing the piano at the increased rate.

B.
the child will go back to playing the piano at the pre-reward rate.

C.
the child will play the piano at a rate less than the pre-reward rate.

D.
the child will initially play the piano at a reduced rate but will eventually go back to the pre-reward rate.

A

Research by M. R. Lepper, D. Greene, and R. E. Nisbett (1973) provided support for the overjustification hypothesis.

a. Incorrect See explanation for response c.
b. Incorrect See explanation for response c.
c. CORRECT The overjustification hypothesis predicts that providing an extrinsic reward for an activity that is intrinsically motivating will reduce intrinsic motivation. Consequently, when the extrinsic reward is discontinued, interest (and participation) in the activity will decline to a level below the pre-extrinsic reward level.
d. Incorrect See explanation for response c.

The correct answer is: the child will play the piano at a rate less than the pre-reward rate.

How well did you know this?
1
Not at all
2
3
4
5
Perfectly
43
Q

In their often-cited study of community mental health services and ethnicity, Sue and his colleagues (1991) found that, with regard to treatment outcomes:
Select one:

A.
Hispanic and African American clients showed substantially less improvement than Anglo and Asian American clients.

B.
Hispanic American clients showed somewhat less improvement than African, Anglo, and Asian American clients.

C.
African American clients showed somewhat less improvement than Hispanic, Anglo, and Asian American clients.

D.
Hispanic, African, and Asian American clients showed less improvement than Anglo American clients.

A

Sue et al. (1991) compared the outcomes of therapy for African, Hispanic, Asian, and Anglo American individuals receiving outpatient mental health services in Los Angeles county.

a. Incorrect See explanation for response c.
b. Incorrect See explanation for response c.
c. CORRECT Sue and his colleagues found that Hispanic American clients had the best outcomes followed by Anglo, Asian, and, lastly, African Americans clients.
d. Incorrect See explanation for response c.

The correct answer is: African American clients showed somewhat less improvement than Hispanic, Anglo, and Asian American clients.

How well did you know this?
1
Not at all
2
3
4
5
Perfectly
44
Q

The results of a study by Diener et al. (1976) found that children who were trick-or-treating on Halloween were more likely to steal candy from a large bowl when the adult was unable to identify individual children when the children were in a large group. This result is explained by which of the following?
Select one:

A.
psychological reactance

B.
inoculation

C.
deindividuation

D.
scapegoating

A

In the study described in this question, children were more likely to steal when they were able to do so anonymously.

a. Incorrect Psychological reactance occurs when a person responds to the request of another person by acting in a way that is opposite to what is being requested because the person feels his/her personal freedom is being threatened.
b. Incorrect In the context of attitude change, inoculation refers to providing a person with arguments and counterarguments prior to hearing a persuasive message in order to reduce the likelihood that the person will be influenced by that message.
c. CORRECT Deindividuation is the increased tendency to act in antisocial ways when it is possible to do so anonymously.
d. Incorrect Scapegoating occurs when a person blames his/her problems or misfortunes on someone else.

The correct answer is: deindividuation

How well did you know this?
1
Not at all
2
3
4
5
Perfectly
45
Q

The research suggests that about _____ percent of children who receive a diagnosis of ADHD continue to meet the diagnostic criteria for the disorder in adolescence.
Select one:

A.
20 to 35

B.
40 to 55

C.
65 to 80

D.
85 to 90

A

Answer C is correct: Estimates vary but most studies indicate that between 65 and 80% of children who receive a diagnosis of ADHD continue to exhibit symptoms that meet the criteria for the diagnosis in adolescence. See, e.g., R. A. Barkley, Taking charge of ADHD: The complete, authoritative guide for parents, New York, Guilford Press, 2013.

Answers A, B, and D: See explanation for answer C.

The correct answer is: 65 to 80

How well did you know this?
1
Not at all
2
3
4
5
Perfectly
46
Q

For adolescents who are hospitalized with Major Depressive Disorder, which of the following is LEAST predictive of future Bipolar Disorder?
Select one:

A.
a family history of a mood disorder

B.
tricyclic-induced hypomania

C.
a gradual onset of depressive symptoms

D.
psychotic features during the depressive episode

A

Answer C is correct: An acute (versus gradual) onset of depressive symptoms in adolescence or early adulthood is associated with an increased risk for Bipolar Disorder.

Answer A: A number of factors (including a family history of a mood disorder) have been found to predict the development of Bipolar Disorder in individuals who have their first episode of Major Depressive Disorder as adolescents or young adults.

Answer B: Tricyclic-induced hypomania has also been linked with the later development of Bipolar Disorder.

Answer D: Psychotic features are also associated with the later development of Bipolar Disorder.

The correct answer is: a gradual onset of depressive symptoms

How well did you know this?
1
Not at all
2
3
4
5
Perfectly
47
Q
A teacher administers a math achievement test to the 25 students in her 6th grade class on the first day of the semester and then again on the last day of the semester to see how much they gained in math ability. To analyze the data she obtains, the teacher should use which of the following statistical tests?
Select one:

A.
t-test for a single sample

B.
t-test for correlated samples

C.
t-test for uncorrelated samples

D.
t-test for multiple means

A

For the exam, you want to know when it is appropriate to use each of the t-tests listed in answers a, b, and c.

a. Incorrect The t-test for a single sample is used to compare an obtained sample mean to a known population mean.
b. CORRECT The t-test for correlated (dependent) samples is used to compare two related means – e.g., means obtained from the same sample at two different times.
c. Incorrect The t-test for uncorrelated (independent) means is used to compare two means obtained from unrelated samples.
d. Incorrect There is no t-test for multiple means.

The correct answer is: t-test for correlated samples

How well did you know this?
1
Not at all
2
3
4
5
Perfectly
48
Q

According to Patterson and his colleagues, aggression in children is linked to the use of coercive discipline by their parents. In turn, the parents’ use of coercive discipline is directly related to:
Select one:

A.
the parents’ misinterpretations of the child’s intentions when he or she misbehaves.

B.
the parents’ personality characteristics and the child’s temperament.

C.
the nature of the attachment between the child and the parent.

D.
the effectiveness of the discipline in stopping the child’s misbehavior.

A

Patterson’s theory regarding coercive family interactions takes into account the reciprocal influences of the parent and child.

a. Incorrect See explanation for response b.
b. CORRECT Patterson (1992) views coercive discipline as being more likely in the presence of stressful life events, certain personality characteristics in the parents, and a temperamentally difficult child.
c. Incorrect See explanation for response b.
d. Incorrect See explanation for response b.

The correct answer is: the parents’ personality characteristics and the child’s temperament.

How well did you know this?
1
Not at all
2
3
4
5
Perfectly
49
Q

An Asian American adult has low identification with both his own culture and the culture of the dominant (Anglo) group. In terms of Berry’s model of acculturation, this is referred to as:
Select one:

A.
separation.

B.
marginalization.

C.
disintegration.

D.
encapsulation.

A

Berry distinguishes between four levels of acculturation based on the individual’s acceptance of his/her own cultural values and the values of the mainstream culture.

a. Incorrect As described by Berry, separation occurs when a person identifies with his/her own culture but rejects the mainstream culture.
b. CORRECT According to Berry, a person is marginalized when he/she does not accept the values, standards, customs, etc. of either culture (i.e., his/her own culture or the culture of the dominant group).
c. Incorrect This is not a term used by Berry.
d. Incorrect This is not a term used by Berry.

The correct answer is: marginalization.

How well did you know this?
1
Not at all
2
3
4
5
Perfectly
50
Q

A 36-year-old woman has extensive deficits in declarative memory as the result of a severe head trauma. This means that she will have trouble recalling:
Select one:

A.
what she ate for dinner the previous evening.

B.
how to ride a bicycle and what she ate for dinner.

C.
the definition of amnesia.

D.
what she ate for dinner and the definition of amnesia.

A

Declarative memory consists of episodic and semantic memory.

a. Incorrect See explanation for response d.
b. Incorrect Riding a bicycle would entail procedural memory.
c. Incorrect See explanation for response d.
d. CORRECT Episodic memory is memory for personally experienced events (e.g., what one had for dinner the previous evening). Semantic memory is memory for factual knowledge (e.g., the definition of amnesia). Extensive impairment in declarative memory would, therefore, involve both types of memory, so this is the best answer.

The correct answer is: what she ate for dinner and the definition of amnesia.

How well did you know this?
1
Not at all
2
3
4
5
Perfectly
51
Q

Tourette’s Disorder, Autism Spectrum Disorder, and Obsessive-Compulsive Disorder have been most consistently linked to pathology in which areas of the brain?
Select one:

A.
amygdala and parietal lobes

B.
thalamus and parietal lobes

C.
basal ganglia and frontal lobes

D.
thalamus and frontal lobes

A

Research using brain imaging techniques has consistently linked abnormalities in certain areas of the brain to Tourette’s Disorder, Autism Spectrum Disorder, and OCD.

a. Incorrect See explanation for response c.
b. Incorrect See explanation for response c.
c. CORRECT Abnormalities in the basal ganglia and frontal lobes have been implicated in all three disorders.
d. Incorrect See explanation for response c.

The correct answer is: basal ganglia and frontal lobes

How well did you know this?
1
Not at all
2
3
4
5
Perfectly
52
Q

In a research study, an investigator interested in the genetic contributors to suicide compares certain demographic, behavioral, and genetic characteristics of a sample of psychiatric patients who attempted suicide in the past month with the characteristics of the first degree and second degree biological relatives of the psychiatric patients and with “healthy controls” (non-patients). The probands in this study are the:
Select one:

A.
psychiatric patients.

B.
third-degree relatives.

C.
first-degree relatives.

D.
first- and second-degree relatives.

A

Answer A is correct: In research, probands (also known as index cases) are the first individuals who are brought to the attention of the investigator - i.e., the people who are manifesting the disease or other characteristics of interest. The psychiatric patients who attempted suicide in the past month are the probands in this study.

The correct answer is: psychiatric patients.

How well did you know this?
1
Not at all
2
3
4
5
Perfectly
53
Q

Which of the following is an example of cultural encapsulation?
Select one:

A.
A client belonging to a minority group cannot separate problems due to individual factors from those related to oppression.

B.
A White male therapist doesn’t recognize that he interprets clients’ problems from his own cultural worldview.

C.
A young woman’s conflict is related to her inability to separate from the values, attitudes, and expectations of her immigrant parents.

D.
A therapist tends to view all problems of minority clients as due to factors inherent to the client’s cultural background.

A

The term cultural encapsulation was used by Wrenn (1985) to describe the characteristics of a therapist who is “encapsulated” within his/her dominant culture.

a. Incorrect See explanation to response b.
b. CORRECT Culturally encapsulated therapists not only view clients’ problems through the filter of their own culture but are also unaware of doing so.
c. Incorrect See explanation for response b.
d. Incorrect See explanation for response b.

The correct answer is: A White male therapist doesn’t recognize that he interprets clients’ problems from his own cultural worldview.

How well did you know this?
1
Not at all
2
3
4
5
Perfectly
54
Q

The final stage in Cross’s (1991) Black Racial Identity Development Model is:
Select one:

A.
internalization.

B.
incorporation.

C.
integrative awareness.

D.
autonomy.

A

The most recent version of Cross’s model distinguishes between four stages of identity development: pre-encounter, encounter, immersion-emersion, and internalization.

a. CORRECT An African-American in the immersion stage has developed a strong ethnic identity but has also resolved global anti-white sentiments and become more bicultural.
b. Incorrect See explanation above.
c. Incorrect See explanation above.
d. Incorrect See explanation above.

The correct answer is: internalization.

How well did you know this?
1
Not at all
2
3
4
5
Perfectly
55
Q

When initially hired by a company, Wally Worker is told that he’ll be receiving regular raises and bonuses as long as his work is satisfactory. Although Wally has received excellent reviews from his supervisor during his two-year period of employment, he has not been given the rewards he was promised. According to expectancy theory, which of the following of Wally’s beliefs will be most negatively affected by these circumstances?
Select one:

A.
valence

B.
autonomy

C.
expectancy

D.
instrumentality

A

Expectancy theory distinguishes between three beliefs that affect motivation – expectancy, instrumentality, and valence.

a. Incorrect Valence refers to the perceived value of the available rewards.
b. Incorrect Autonomy is not part of expectancy theory.
c. Incorrect Expectancy is the belief that one’s effort will result in performance.
d. CORRECT Instrumentality refers to the belief that one’s performance will be rewarded.

The correct answer is: instrumentality

How well did you know this?
1
Not at all
2
3
4
5
Perfectly
56
Q

Herzberg’s two-factor theory predicts that, if a worker’s job is redesigned so that she has less autonomy and responsibility but is also given a raise in salary, the worker will be:
Select one:

A.
dissatisfied.

B.
neither satisfied nor dissatisfied.

C.
satisfied.

D.
both satisfied and dissatisfied.

A

Herzberg’s two-factor theory distinguishes between hygiene and motivator factors.

a. Incorrect See explanation for response b.
b. CORRECT Herzberg’s theory predicts that motivator factors produce satisfaction when they are present but have no effect on satisfaction if they are not present. In contrast, hygiene factors produce dissatisfaction when they are not adequate but have no effect on satisfaction when they’re adequate. Consequently, if the worker’s job is redesigned so that it produces few motivator factors (e.g., less autonomy and responsibility) and more adequate hygiene factors (pay), the worker will be neither satisfied nor dissatisfied.
c. Incorrect See explanation for response b.
d. Incorrect See explanation for response b.

The correct answer is: neither satisfied nor dissatisfied.

How well did you know this?
1
Not at all
2
3
4
5
Perfectly
57
Q

A young man is found guilty of driving while intoxicated and is required by the court to receive in-patient treatment at an alcohol rehabilitation center. As a therapist working at the rehabilitation center, you should be aware that:
Select one:

A.
it is not necessary to have the man sign a waiver of confidentiality since his treatment is court-ordered.

B.
it is not necessary to have the man sign a waiver of confideniality because he has apparently voluntarily chosen treatment rather than some other form of punishment.

C.
it is not necessary to have the man sign a waiver of confidentiality since no information should be given to the court in this situation under any circumstances.

D.
it is necessary to have the man sign a waiver of confidentiality before releasing any information to the court.

A

The best approach to answering a question on informed consents and waivers of confidentiality is to choose the most conservative answer.

a. Incorrect See explanation for response d.
b. Incorrect See explanation for response d.
c. Incorrect See explanation for response d.
d. CORRECT Although the court has ordered the treatment, the client’s right to confidentiality is not waived. In fact, a court order is likely to be required for the release of information to it, and the information that must be released would likely be limited to objective data (see, e.g., L. C. Sobell and M. B. Sobell, Client rights in alcohol treatment programs, In. G. T. Hannah, et al. (eds.), Preservation of Client Rights, New York: The Free Press, 1981).

The correct answer is: it is necessary to have the man sign a waiver of confidentiality before releasing any information to the court.

How well did you know this?
1
Not at all
2
3
4
5
Perfectly
58
Q

There is evidence that stimulus exposure and antidepressants that increase serotonin levels both alleviate the symptoms of Obsessive-Compulsive Disorder by reducing activity in the:
Select one:

A.
locus coeruleus.

B.
caudate nucleus.

C.
reticular activating system.

D.
inferior colliculus.

A

Answer B is correct: L. R. Baxter and associates found evidence that behavioral and drug treatments for OCD both reduce activity in the caudate nucleus, which is involved in converting sensory input into cognition and action and is over-active in people with OCD (Caudate glucose metabolic rate changes with both drug and behavior therapy for obsessive-compulsive disorder, Archives of General Psychiatry, 49, 681-689, 1992).

Answers A, C, and D: See explanation for answer B.

The correct answer is: caudate nucleus.

How well did you know this?
1
Not at all
2
3
4
5
Perfectly
59
Q

A psychophysicist is measuring “just noticeable differences” while investigating the relationship between changes in the intensity of light and her research participants’ perceptions of those changes. The measuring scale being used has:
Select one:

A.
physically equal intervals.

B.
psychologically equal intervals.

C.
physically and psychologically equal intervals.

D.
physically and psychologically unequal intervals.

A

Psychophysics is the study of the relationship between stimulus magnitude and perception of that stimulus.

a. Incorrect See explanation for response b.
b. CORRECT Psychophysicists use just noticeable differences (JNDs) and similar measurements to study the relationship between changes in physical stimuli and the psychological responses to those changes. JNDs are considered to be equal; however, the corresponding physical stimuli usually are not. For example, it may take only the addition of one pound to notice a difference when you start with ten pounds but the addition of ten pounds to notice a difference when you start with 100 pounds. In each case, the JND is equal to one, but the physical differences in weight are not the same.
c. Incorrect See explanation for response b.
d. Incorrect See explanation for response b.

The correct answer is: psychologically equal intervals.

How well did you know this?
1
Not at all
2
3
4
5
Perfectly
60
Q

Dr. Ralph R. is presenting a workshop on reducing anxiety and tells the editor of a community newsletter that she can participate for free if she puts an article about the workshop in the newsletter. From an ethical perspective, this arrangement is:
Select one:

A.
acceptable as long as the arrangement doesn’t adversely affect the editor’s ability to benefit from the workshop.

B.
acceptable as long as the article provides accurate information about the workshop.

C.
unacceptable because it constitutes bartering of services, which are prohibited by ethical guidelines.

D.
unacceptable because psychologists are prohibited by ethical guidelines from compensating employees of any communication media for publicity in a news item.

A

This issue is explicitly addressed in Standard 5.02(b) of the APA’s Ethics Code and indirectly addressed in Principle III.31 of the Canadian Code of Ethics.

a. Incorrect See explanation for response b.
b. Incorrect See explanation for response b.
c. Incorrect Bartering of services is not necessarily prohibited by the Ethics Code but the specific type of exchange described in this question is prohibited, which is why answer d is the best response.
d. CORRECT Standard 5.02(b) prohibits psychologists from compensating employees of the press, radio, or other commmunication media for providing publicity in a news items about the psychologist or his/her work.

The correct answer is: unacceptable because psychologists are prohibited by ethical guidelines from compensating employees of any communication media for publicity in a news item.

How well did you know this?
1
Not at all
2
3
4
5
Perfectly
61
Q

One problem with a decentralized communication network is that:
Select one:

A.
it is associated with low levels of employee morale.

B.
it can lead to the development of autocratic leadership.

C.
it can confuse the situation when the task is simple.

D.
it is effective only when the company’s organizational structure is also decentralized.

A

In a decentralized communication network, all people in the system can communicate freely with each other.

a. Incorrect This is more likely to occur with a centralized communication network.
b. Incorrect This is also more characteristic of a centralized communication network.
c. CORRECT For simple tasks (e.g., an automated production line), centralized communication networks tend to be more effective.
d. Incorrect This is not necessarily true. The most effective communication network may not necessarily be the one that matches the organizational structure.

The correct answer is: it can confuse the situation when the task is simple.

How well did you know this?
1
Not at all
2
3
4
5
Perfectly
62
Q

The developer of a new predictor realizes that the predictor’s criterion-related validity coefficient has been affected by “criterion contamination.” If the developer conducts a new validity study and eliminates the effects of criterion contamination, he/she can expect which of the following to occur?
Select one:

A.
The new validity coefficient will be larger in magnitude than the original coefficient.

B.
The new validity coefficient will be smaller in magnitude than the original coefficient.

C.
The new validity coefficient will be either larger or smaller in magnitude than the original coefficient.

D.
The new validity coefficient will be neither larger nor smaller in magnitude than the original coefficient.

A

When a rater’s knowledge of the ratees’ predictor scores affects his/her ratings of the ratees on the criterion measure (i.e., when the rater rates a ratee high on the criterion because the ratee has scored high on the predictor and vice-versa), an artificially strong relationship is created between the predictor and criterion.

a. Incorrect See explanation for response b.
b. CORRECT Criterion contamination artificially inflates the correlation coefficient. Therefore, the elimination of criterion contamination would lower the coefficient.
c. Incorrect See explanation for response b.
d. Incorrect See explanation for response b.

The correct answer is: The new validity coefficient will be smaller in magnitude than the original coefficient.

How well did you know this?
1
Not at all
2
3
4
5
Perfectly
63
Q

In a multitrait-multimethod matrix, the __________ coefficients are correlation coefficients that provide information about a test’s convergent validity.
Select one:

A.
heterotrait-heteromethod

B.
heterotrait-monomethod

C.
monotrait-heteromethod

D.
monotrait-monomethod

A

A multitrait-multimethod matrix provides information about a test’s convergent and divergent (discriminant) validity.

a. Incorrect A heterotrait-heteromethod coefficient provides information about a test’s divergent validity.
b. Incorrect A heterotrait-monomethod coefficient also provides information about a test’s divergent validity.
c. CORRECT A monotrait-heteromethod coefficient indicates the correlation between the same trait being measured with different methods – i.e., it provides information about a test’s convergent validity.
d. Incorrect The monotrait-monomethod coefficient provides information about a test’s reliability.

The correct answer is: monotrait-heteromethod

How well did you know this?
1
Not at all
2
3
4
5
Perfectly
64
Q

A brief period of electrical stimulation of the hippocampus enhances nerve cell electrical activity in that area of the brain for minutes to hours. This is referred to as:
Select one:

A.
graded potentiation.

B.
hyperpolarization.

C.
absolute refractory period.

D.
long-term potentiation.

A

Interestingly, the phenomenon described in this question not only occurs in the living brain but can also be induced in cells that have been removed from the brain.

a. Incorrect Graded potentiation is a made-up term.
b. Incorrect Hyperpolarization refers to a state of inhibition.
c. Incorrect During the absolute refractory period, a cell cannot “fire” regardless of the amount of stimulation.
d. CORRECT This is the name given to the phenomenon described in this question. Long-term potentiation (LTP) is believed to underlie certain types of learning and memory.

The correct answer is: long-term potentiation.

How well did you know this?
1
Not at all
2
3
4
5
Perfectly
65
Q

Cognitive preparation, skills acquisition and rehearsal, and application and follow-through are the three phases of which of the following interventions?
Select one:

A.
self-control therapy

B.
self-instructional training

C.
attribution retraining

D.
stress inoculation training

A

For the exam, you want to be familiar with the interventions listed in the answers to this question. These are described in the Learning Theory chapter of the written study materials.

a. Incorrect See explanation for response d.
b. Incorrect See explanation for response d.
c. Incorrect See explanation for response d.
d. CORRECT Stress inoculation training was designed to help people deal with stress by improving their coping skills. It involves the stages listed in this question (although the stages are sometimes identified using slightly different names).

The correct answer is: stress inoculation training

How well did you know this?
1
Not at all
2
3
4
5
Perfectly
66
Q

According to Ridley (1984), “cultural paranoia” is:
Select one:

A.
a sign of depression, anxiety, or a psychotic disorder.

B.
a healthy reaction to racism.

C.
a manifestation of racist beliefs.

D.
a sign of internalized oppression.

A

Ridley (1984) was interested in nondisclosure by African American therapy clients and described it as the result of two types of paranoia - cultural and functional.

a. Incorrect See explanation for response b.
b. CORRECT According to Ridley, cultural paranoia is a healthy, adaptive reaction to racism. One result of cultural paranoia is that African American clients may mistrust a White therapist and, as a result, be unwilling to self-disclose in therapy.
c. Incorrect See explanation for response b.
d. Incorrect See explanation for response b.

The correct answer is: a healthy reaction to racism.

How well did you know this?
1
Not at all
2
3
4
5
Perfectly
67
Q

A solution-focused therapist is working with a husband and wife whose conversations frequently escalate to bitter fights. The therapist will most likely:
Select one:

A.
reduce tension during therapy sessions by having the husband and wife talk directly to her rather than to each other.

B.
instruct the couple to take a ten-minute “time-out” whenever they begin to argue at home.

C.
ask the couple if they can remember a time when they were able to talk for an extended period without arguing.

D.
ask the couple what rewards they derive from arguing.

A

As its name implies, solution-focused therapy focuses on solutions rather than problems.

a. Incorrect This sounds more like the approach a Bowenian family therapist would adopt.
b. Incorrect This isn’t characteristic of the solution-focused approach.
c. CORRECT Solution-focused therapists use “exceptions” to help identify solutions to a client’s problems. An exception is a time when the problem did not exist or existed in a less intense form.
d. Incorrect This is not characteristic of this approach.

The correct answer is: ask the couple if they can remember a time when they were able to talk for an extended period without arguing.

How well did you know this?
1
Not at all
2
3
4
5
Perfectly
68
Q

Factor analyses of Raven’s Progressive Matrices has found that it is a reliable measure of which of the following?
Select one:

A.
Spearman’s “g”

B.
Horn and Cattell’s fluid and crystallized intelligence

C.
Luria’s simultaneous and sequential processing

D.
Guilford’s convergent and divergent thinking

A

Knowing that Raven’s Progressive Matrices is a nonverbal measure of intelligence may have helped you identify the correct answer to this question.

a. CORRECT Raven’s Progressive Matrices was originally designed to be a measure of Spearman’s “g” factor and its usefulness for this purpose has been confirmed by a number of factor analytic studies.
b. Incorrect See explanation above.
c. Incorrect See explanation above.
d. Incorrect See explanation above.

The correct answer is: Spearman’s “g”

How well did you know this?
1
Not at all
2
3
4
5
Perfectly
69
Q

Carstensen’s (1993) socioemotional activity theory predicts that changes in ________ over the lifespan are related to changes in time perspective.
Select one:

A.
social self-esteem

B.
personality traits

C.
emotional reactivity

D.
social goals

A

Socioemotional activity theory addresses the motivational processes that underlie changes in the quality and quantity of social relationships over the lifespan.

a. Incorrect See explanation for response d.
b. Incorrect See explanation for response d.
c. Incorrect See explanation for response d.
d. CORRECT According to Carstenson (1993), time perspective plays a key role in determining social goals – i.e., people with a limited time perspective focus on the emotional aspects of relationships while those with an unlimited time perspective focus more on informational aspects.

The correct answer is: social goals

How well did you know this?
1
Not at all
2
3
4
5
Perfectly
70
Q

__________ is a neuroleptic-induced movement disorder that is characterized by a subjective feeling of restlessness accompanied by stereotypical motor movements such as fidgeting, pacing, or foot tapping.
Select one:

A.
Dystonia

B.
Akathisia

C.
Akinesia

D.
Parkinsonism

A

For the exam, you want to be familiar with the characteristics of the four conditions listed in the answers to this question.

a. Incorrect Dystonia is characterized by “abnormal and prolonged contraction of the muscles of the eyes …, head, neck …, limbs, or truck” (DSM-5, p. 711).
b. CORRECT The symptoms described in this question are characteristic of akathisia, which usually develops within a few weeks after starting or increasing the dose of a neuroleptic drug.
c. Incorrect Akinesia is one of the symptoms of neuroleptic-induced parkinsonism and involves a slowing or lack of normal motor function
d. Incorrect Parkinsonism involves tremor, muscle rigidity, and akinesia.

The correct answer is: Akathisia

How well did you know this?
1
Not at all
2
3
4
5
Perfectly
71
Q

Which of the following best describes current views regarding the use of drugs to treat delirium in older adults?
Select one:

A.
Antipsychotic drugs are contraindicated for older adults because of their ineffectiveness.

B.
Newer (atypical) antipsychotic drugs are substantially more effective than haloperidol.

C.
Haloperidol is effective but should be administered initially at a low dose and then gradually increased if necessary.

D.
Haloperidol is effective only when administered in conjunction with a benzodiazepine.

A

The American Psychiatric Association’s guidelines for the treatment of delirium identify haloperidol as the drug-of-choice for the treatment of this disorder, and the research has confirmed that it is as effective for older adults as it is for younger adults.

a. Incorrect The research has shown that haloperidol is about equally effective for reducing agitation, confusion, hallucinations, and certain other symptoms in older and younger adults.
b. Incorrect Although atypical antipsychotic drugs (especially risperidone) have been found useful for treating delirium, the research has not found them to be “substantially more effective” than haloperidol.
c. CORRECT Because older adults are more sensitive to the therapeutic and side effects of antipsychotics (as well as a number of other drugs), the experts recommend initiating treatment with a low dose and then increasing the dose if necessary to achieve the desired effects.
d. Incorrect The benzodiazepines are a treatment-of-choice in some cases of delirium (e.g., when it is caused by alcohol or sedative-hypnotic withdrawal), but they are not ordinarily administered in conjunction with haloperidol.

The correct answer is: Haloperidol is effective but should be administered initially at a low dose and then gradually increased if necessary.

How well did you know this?
1
Not at all
2
3
4
5
Perfectly
72
Q

A woman’s clothing store hires only females as sales clerks because one of the clerks’ tasks is to assist women in the store’s dressing room. In this case, the store would use which of the following standards to justify hiring only women as sales clerks?
Select one:

A.
quid pro quo

B.
business necessity

C.
bona fide occupational qualification

D.
systematic discrimination

A

In the situation described in the question, gender is a necessary qualification for the job.

a. Incorrect Quid pro quo is a term associated with sexual harassment and occurs when a hiring or employment decision is based on the person’s submission to or rejection of sexual advances.
b. Incorrect Business necessity is a defense to a charge of adverse impact and refers to using a discriminatory hiring practice on the ground that it is job related and necessary for the safe and efficient operation of the business. For example, considering only applicants with specific educational qualifications is a business necessity when those qualifications are necessary to perform the job at a minimum level of competence.
c. CORRECT Bona fide occupational qualification (BFOQ) is another defense to a charge of adverse impact and applies when the characteristic is an essential determinant of the genuineness of the job or must be taken into consideration to maintain community standards of morality or propriety. Gender is a BFOQ for the job described in this question.
d. Incorrect Systematic discrimination refers to a pattern of discrimination at a particular place of employment that is due to pervasive, interrelated policies and procedures.

The correct answer is: bona fide occupational qualification

How well did you know this?
1
Not at all
2
3
4
5
Perfectly
73
Q

The predictions of goal-setting theory appear to be most accurate for:
Select one:

A.
easy tasks and people high in need for achievement.

B.
easy tasks and people low in need for achievement.

C.
complex tasks and people high in need for achievement.

D.
complex tasks and people low in need for achievement.

A

While Edwin Locke (1968) considered his goal-setting theory to be applicable to all situations, the research suggests that it may apply better to certain types of tasks and certain people.

a. CORRECT There is some evidence that the predictions of goal setting theory are more accurate for simple tasks and for people who are high in need for achievement (who are more likely to commit to goals).
b. Incorrect See explanation for response a.
c. Incorrect See explanation for response a.
d. Incorrect See explanation for response a.

The correct answer is: easy tasks and people high in need for achievement.

How well did you know this?
1
Not at all
2
3
4
5
Perfectly
74
Q

An African American client in the dissonance stage of Atkinson, Morten, and Sue’s Racial/Cultural Identity Development Model will exhibit which of the following?
Select one:

A.
self-deprecating and same-group deprecating attitudes

B.
self-appreciating but dominant-group deprecating attitudes

C.
a conflict between self-appreciating and self-deprecating attitudes and between same-group appreciating and same-group deprecating attitudes

D.
concern over the basis of his/her self-appreciating and unequivocal same-group appreciating attitudes

A

The stages of the Atkinson, Morten, and Sue model are each characterized by different attitudes toward oneself, one’s own (same) minority group, other minority groups, and the majority (mainstream) group.

a. Incorrect This is characteristic of the conformity stage (stage 1).
b. Incorrect This is characteristic of the resistance and immersion stage (stage 3).
c. CORRECT Not surprisingly, a person in the dissonance stage (stage 2) will be conflicted about his/her attitudes.
d. Incorrect This is characteristic of the introspection stage (stage 4).

The correct answer is: a conflict between self-appreciating and self-deprecating attitudes and between same-group appreciating and same-group deprecating attitudes

How well did you know this?
1
Not at all
2
3
4
5
Perfectly
75
Q

The DSM-5 requires that which of the following be ruled out as the cause of an individual’s symptoms before assigning a diagnosis of a sexual dysfunction?
Select one:

A.
other mental disorders, physiological/medical factors, and partner factors

B.
a nonsexual mental disorder, severe relationship distress, and the effects of a substance, medication, or other medical condition

C.
a nonsexual mental disorder, the effects of a substance/medication, and a lack of information about sexuality

D.
other mental disorder/condition, the effects of a substance/medication, and psychosocial factors

A

Answer B is correct: The following is included as a diagnostic criterion for a DSM-5 diagnosis of Erectile Disorder, Premature Ejaculation, and other sexual dysfunctions: “The sexual dysfunction is not better explained by a nonsexual mental disorder or as a consequence of severe relationship distress or other significant stressors and is not attributable to the effects of a substance/medication or another medical condition.”

Answers A, C, and D: See explanation above.

The correct answer is: a nonsexual mental disorder, severe relationship distress, and the effects of a substance, medication, or other medical condition

How well did you know this?
1
Not at all
2
3
4
5
Perfectly
76
Q

A practitioner of cognitive therapy is most likely to conceptualize a personality disorder as a consequence of:
Select one:

A.
impulsive action tendencies.

B.
cognitive enactments.

C.
maladaptive life scripts.

D.
compelling schemata.

A

Even if you are unfamiliar with the cognitive view of the personality disorders, your familiarity with cognitive therapy in general should have helped you identify the correct answer to this question.

a. Incorrect See explanation for response d.
b. Incorrect See explanation for response d.
c. Incorrect See explanation for response d.
d. CORRECT Although the word “compelling” might be unfamiliar, the word “schemata” is one you want to have associated cognitive therapy. According to A. Freeman and R. C. Leaf, people with personality disorders are “generally governed by very compelling schemata,” which are not easily challenged or surrendered” [Cognitive therapy applied to personality disorders, in A. Freeman, et al. (eds.), Comprehensive Handbook of Cognitive Therapy, New York: Plenum Press, 1989].

The correct answer is: compelling schemata.

How well did you know this?
1
Not at all
2
3
4
5
Perfectly
77
Q

Research on which of the following found that, over time, people tend to remember a message but forget the messenger?
Select one:

A.
psychological reactance

B.
gain-loss effect

C.
sleeper effect

D.
attitude inoculation

A

For the exam, you want to be familiar with the phenomena listed in the answers to these questions. Information about them is provided in the Social Psychology chapter of the written study materials.

a. Incorrect See explanation for response c.
b. Incorrect See explanation for response c.
c. CORRECT Research on the sleeper effect found that the initial difference in the persuasiveness of high- and low-credible communicators disappears over time apparently because people tend to forget the source of the message.
d. Incorrect See explanation for response c.

The correct answer is: sleeper effect

How well did you know this?
1
Not at all
2
3
4
5
Perfectly
78
Q

Tiedeman and O’Hara’s (1963) model of career development emphasizes which of the following?
Select one:

A.
basic interests

B.
basic instinctual needs

C.
vocational identity

D.
locus of control

A

Tiedeman and O’Hara (1963) viewed career development as a process involving the acquisition of a personal vocational identity.

a. Incorrect See explanation for response c.
b. Incorrect See explanation for response c.
c. CORRECT According to these authors, the achievement of a personal vocational identity involves balancing integration (being part of a career field) with differentiation (retaining individuality and uniqueness).
d. Incorrect See explanation for response c.

The correct answer is: vocational identity

How well did you know this?
1
Not at all
2
3
4
5
Perfectly
79
Q

Kohlberg argued that there is a monotonic relationship between moral judgment and moral action and proposed that, as one moves from a lower to a higher stage of moral development:
Select one:

A.
the range of possible moral actions becomes narrower and the individual assumes greater responsibility for relating his/her judgments to actions.

B.
the range of possible moral actions becomes broader and the individual assumes greater responsibility for relating his/her judgments to actions.

C.
the range of possible moral actions remains the same but the individual assumes greater responsibility for relating his/her judgments to actions.

D.
the range of possible moral actions becomes narrower but the individual’s sense of responsibility for relating his/her judgments to actions remains about the same.

A

Although Kohlberg believed that there is a positive relationship between moral judgment and moral action, he proposed that this relationship is much stronger at higher levels of moral development.

a. CORRECT The stronger relationship between judgment and action at higher stages is attributable to two factors: first, the range of actions suggested by the individual’s judgment narrows; and, second, the individual experiences an increasing sense of responsibility to act.
b. Incorrect See explanation for response a.
c. Incorrect See explanation for response a.
d. Incorrect See explanation for response a.

The correct answer is: the range of possible moral actions becomes narrower and the individual assumes greater responsibility for relating his/her judgments to actions.

How well did you know this?
1
Not at all
2
3
4
5
Perfectly
80
Q

As a treatment for Borderline Personality Disorder, Linehan’s (1987) dialectical behavior therapy emphasizes which of the following?
Select one:

A.
altering the client’s dysfunctional behaviors using modeling and behavior rehearsal

B.
using family therapy to alter the client’s social and emotional environment

C.
improving the client’s problem-solving and social skills

D.
strengthening the client’s weak ego

A

Answer C is correct: Knowing that dialectical behavior therapy (DBT) includes group skills training aimed at improving the client’s social and coping skills would have helped you identify this as the correct answer to this question.

Answers A and B: See explanation for answer C.

Answer D: Strengthening the client’s weak ego is a goal of object relations psychotherapy for Borderline Personality Disorder.

The correct answer is: improving the client’s problem-solving and social skills

How well did you know this?
1
Not at all
2
3
4
5
Perfectly
81
Q

B. F. Skinner attributed which of the following to accidental, noncontingent reinforcement?
Select one:

A.
experimental neurosis

B.
superstitious behavior

C.
overshadowing

D.
satiation

A

For the exam, you want to be familiar with the terms listed in the answers to this question. These are described in the Learning Theory chapter of the written study materials.

a. Incorrect See explanation for response b.
b. CORRECT Skinner found that his animal subjects would repeat behaviors that were accidentally reinforced and proposed that noncontingent reinforcement may account for the acquisition of superstitious behaviors in humans.
c. Incorrect See explanation for response b.
d. Incorrect See explanation for response b.

The correct answer is: superstitious behavior

How well did you know this?
1
Not at all
2
3
4
5
Perfectly
82
Q

Several authorities have noted that traditional forms of psychotherapy reflect a Eurocentric (Western) perspective that emphasizes:
Select one:

A.
a concrete non-linear approach.

B.
a linear cause-effect approach.

C.
a reciprocal (interactive) approach.

D.
a directive behavioral approach.

A

This issue has been addressed by several experts interested in family therapy or cross-cultural counseling.

a. Incorrect See explanation for response b.
b. CORRECT The Eurocentric perspective emphasizes one-on-one problem-solving and adopts an atomistic, linear, and reductionist cause-effect approach.
c. Incorrect See explanation for response b.
d. Incorrect See explanation for response b.

The correct answer is: a linear cause-effect approach.

How well did you know this?
1
Not at all
2
3
4
5
Perfectly
83
Q

Which of the following types of biofeedback is likely to be most effective for reducing migraine headaches?
Select one:

A.
skin conductance

B.
blood volume pulse

C.
EMG (electromyography)

D.
EEG (electroencephalograph)

A

Several types of biofeedback have been evaluated as treatments for migraine headaches.

a. Incorrect Skin conductance biofeedback provides information on autonomic arousal and is used primarily to reduce stress and anxiety reactions.
b. CORRECT Several studies indicate that blood volume pulse biofeedback is as effective as or more effective than skin temperature biofeedback for treating migraine headaches. Blood volume pulse is measured using a photoplethysmograph (PPG) that monitors change in blood flow in the blood vessels beneath the skin. See, e.g., Y. Nestoriuc & A. Martin, Efficacy of biofeedback for migraine: A meta-analysis, Pain, 128(1-2), 111-127, 2007.
c. Incorrect EMG biofeedback is useful for muscle tension headaches but has not been found to be more effective than blood volume pulse biofeedback for treating migraine headaches.
d. Incorrect EEG biofeedback provides information on brain activity and is not used as a treatment for migraine headaches.

The correct answer is: blood volume pulse

How well did you know this?
1
Not at all
2
3
4
5
Perfectly
84
Q

To assess the plausibility of a causal theory about the relationships among a set of variables, you would use which of the following?
Select one:

A.
principal components analysis

B.
structural equation modeling

C.
canonical correlation

D.
cluster analysis

A

Of the four multivariate techniques listed in the answers to this question, only one is used to test a causal model or theory.

a. Incorrect Principal components analysis is similar to factor analysis and is used to identify a set of variables (components) that explains the variability in a set of tests.
b. CORRECT Structural equation modeling is used to test cause-and-effect relationships among a set of variables.
c. Incorrect Canonical correlation is used to determine the degree of association between two more more predictors and two or more criteria.
d. Incorrect As its name implies, cluster analysis is used to place variables into clusters (categories).

The correct answer is: structural equation modeling

How well did you know this?
1
Not at all
2
3
4
5
Perfectly
85
Q

According to Fiedler’s contingency theory of leadership, a task-motivated (“low LPC”) leader is most effective when his or her relations with subordinates are:
Select one:

A.
very good.

B.
very poor.

C.
very good or very poor.

D.
moderate.

A

Fiedler’s theory predicts that task-motivated (low LPC) leaders and relationship-motivated (high LPC) leaders are most effective in different circumstances. One circumstance that determines the best leadership style is the nature of leader-member relations.

a. Incorrect See explanation for response c.
b. Incorrect See explanation for response c.
c. CORRECT Task-motivated leaders are most effective in extreme situations - that is, when leader-member relations are either very good or very poor.
d. Incorrect Relationship-motivated leaders are most effective in less extreme situations - e.g., when leader-member relations are moderate (between good and poor).

The correct answer is: very good or very poor.

How well did you know this?
1
Not at all
2
3
4
5
Perfectly
86
Q

The research has consistently linked increasing age with decreases in scores on measures of ____________ and suggests that these decreases may be related to declines in the efficiency of working memory.
Select one:

A.
verbal comprehension

B.
nonverbal comprehension

C.
crystallized intelligence

D.
fluid intelligence

A

For the exam, you want to be familiar with the difference between fluid and crystallized intelligence, including differences in their vulnerability to age-related decline.

a. Incorrect See explanation for response b.
b. Incorrect See explanation for response b.
c. Incorrect See explanation for response b.
d. CORRECT The research has generally found that scores on measures of crystallized intelligence do not begin to decline until the mid-50s or later, while scores on measures of fluid intelligence begin to decline in early adulthood. In addition, there is evidence that decreases in fluid intelligence are attributable to declines in working memory.

The correct answer is: fluid intelligence

How well did you know this?
1
Not at all
2
3
4
5
Perfectly
87
Q

A meta-analysis of the research by Baltes and his colleagues (1999) found that instituting flextime in an organization is likely to have the LEAST beneficial effects on which of the following?
Select one:

A.
absenteeism

B.
self-rated performance

C.
satisfaction with work schedule

D.
overall job satisfaction

A

Unfortunately, the research on the effects of flextime and other alternative work schedules has produced inconsistent results. However, this question is asking about the results of a specific meta-analysis of the research.

a. Incorrect See explanation for response b.
b. CORRECT Baltes et al. found that flextime had a substantial positive effects on absenteeism, satisfaction with the work schedule, and overall job satsifaction but had little effect on self-rated performance. For additional information on the Baltes et al. meta-analysis, see the Industrial-Organizational Psychology chapter of the written study materials.
c. Incorrect See explanation for response b.
d. Incorrect See explanation for response b.

The correct answer is: self-rated performance

How well did you know this?
1
Not at all
2
3
4
5
Perfectly
88
Q

As a diathesis-stress model, Beck’s cognitive theory predicts that which of the following serves as the diathesis in the development of depressive symptomology?
Select one:

A.
dysfunctional schemas

B.
a genetic predisposition

C.
exposure to a negative event

D.
faulty attributions

A

Diathesis-stress models describe illness as the result of a combination of vulnerability (diathesis) and stress, with the nature of the vulnerability depending on the type of theory. Not surprisingly, Beck’s cognitive theory of depression identifies cognitive factors as the source of vulnerability.

a. CORRECT Beck’s cognitive diathesis-stress model of depression proposes that certain people have depressogenic schemas that make them vulnerable to experiencing depression when exposed to a negative life event.
b. Incorrect Beck’s cognitive theory focuses on cognitive, not biological, factors.
c. Incorrect Exposure to a negative event is the “stress” in Beck’s cognitive diathesis-stress model.
d. Incorrect Certain types of attributions - i.e., internal, stable, and global attributions - for negative outcomes are identified as the diathesis in Abramson, Metalsky, and Alloy’s (1989) hopelessness model of depression. Although their theory is a cognitive theory, this question is asking about Beck’s cognitive theory, so answer a is the best answer to this question.

The correct answer is: dysfunctional schemas

How well did you know this?
1
Not at all
2
3
4
5
Perfectly
89
Q

In the context of test validity, ________ is said to occur when a criterion-related validity coefficient is smaller in size for a cross-validation sample than it was for the initial sample.
Select one:

A.
criterion deficiency

B.
contracture

C.
shrinkage

D.
attenuation

A

Cross-validation involves re-assessing a test’s criterion-related validity on a new sample to check the generalizability of the original validity coefficient.

a. Incorrect See explanation for response c.
b. Incorrect See explanation for response c.
c. CORRECT The validity coefficient usually “shrinks” on cross-validation because the chance factors operating in the original sample are not all present in the cross-validation sample.
d. Incorrect See explanation for response c.

The correct answer is: shrinkage

How well did you know this?
1
Not at all
2
3
4
5
Perfectly
90
Q

When the item discrimination index (D) equals _____, this means that all of the examinees in the upper-scoring group and none of the examinees in the lower-scoring group answered the item correctly.
Select one:

A.
50

B.
1

C.
-1

D.
0

A

Answer B is correct. The item discrimination index ranges in value from -1.0 to +1.0. An item discrimination index of +1.0 indicates maximum discrimination between high- and low-scoring groups, with all examinees in the upper group answering the item correctly and none of the examinees in the lower group answering the item correctly.

The correct answer is: 1

How well did you know this?
1
Not at all
2
3
4
5
Perfectly
91
Q

According to __________, the innate drive to develop attachment to one’s caregivers developed as a survival mechanism during the course of human evolution.
Select one:

A.
existential theory

B.
object relations theory

C.
Jung

D.
Adler

A

Early relationships between an infant and his/her caregivers are considered critical in many theories of personality development, but only one of the approaches listed explicitly describes attachment as an evolutionary phenomenon that developed as a survival mechanism.

a. Incorrect See explanation for response b.
b. CORRECT According to object relations theory, the tendency toward attachment is an innate capacity that developed through evolution as a survival mechanism; and problems related to attachment in infancy have negative consequences for adult personality.
c. Incorrect See explanation for response b.
d. Incorrect See explanation for response b.

The correct answer is: object relations theory

92
Q

When creating an acrostic:
Select one:

A.
a sentence or rhyme is formed using words that begin with the first letter of the words to be memorized.

B.
a word or pronounceable sequence of letters is created from the first letter of the words to be memorized.

C.
a word or phrase is constructed from another word by rearranging the letters of that word.

D.
an image is created from the images of two or more words.

A

For the exam, you want to be familiar with an acrostic and the other mnemonic devices described in the Learning Theory chapter of the written study materials.

a. CORRECT An acrostic is a phrase that is formed from words that begin with the first letter of each word to be memorized. “See Piaget creep forward” is an acrostic for Piaget’s four stages of cognitive development (sensorimotor, preoperational, concrete operational, and formal operational).
b. Incorrect This describes an acronym, which is a type of word play rather than a mnemonic device.
c. Incorrect This describes an anagram.
d. Incorrect This describes the keyword method.

The correct answer is: a sentence or rhyme is formed using words that begin with the first letter of the words to be memorized.

93
Q

Squaring the multiple correlation coefficient (R) provides a measure of:
Select one:

A.
shared variability.

B.
true score variability.

C.
average variability.

D.
extraneous variability.

A

The multiple correlation coefficient, like a bivariate correlation coefficient, can be squared to obtain a coefficient of determination.

a. CORRECT The squared multiple correlation coefficient indicates the amount of variability in the criterion that is shared with the weighted combination of the predictors.
b. Incorrect See explanation above.
c. Incorrect See explanation above.
d. Incorrect See explanation above.

The correct answer is: shared variability.

94
Q

A number of neuropsychologists interested in constructive memory have attempted to identify the areas of the brain that are involved in false recollections. Their research has most consistently linked false recollection to lesions in the:
Select one:

A.
frontal lobe.

B.
temporal lobe.

C.
occipital lobe.

D.
parietal lobe.

A

The current view of memory is that it does not involve a literal recollection of past events but instead a constructive process that is susceptible to errors, distortions, and other factors that limit the accuracy of memories.

a. CORRECT The research has most consistently linked false recollections to lesions in the frontal lobes, especially the prefrontal cortex. See, e.g., D. L. Schacter and S. D. Slotnick, The cognitive neuroscience of memory distortion, Neuron, 44, 149-160, 2004.
b. Incorrect See explanation above.
c. Incorrect See explanation above.
d. Incorrect See explanation above.

The correct answer is: frontal lobe.

95
Q

Research has found that which of the following is first depth cue to emerge and is apparent by 3 to 4 weeks of age?
Select one:

A.
binocular

B.
motion

C.
pictorial

D.
contrast sensitivity

A

Sensitivity to depth cues emerges in a predictable sequence – motion, binocular, and pictorial.

a. Incorrect See explanation for response b.
b. CORRECT Sensitivity to motion (kinetic) cues is apparent by 3 to 4 weeks of age when infants begin to blink their eyes defensively in response to an object being moved toward their face.
c. Incorrect See explanation for response b.
d. Incorrect See explanation for response b.

The correct answer is: motion

96
Q

Problems involving sensation in the inner forearm and the ring and little fingers are most likely due to spinal injury at the ______ level.
Select one:

A.
L1

B.
S1 to S4

C.
T2 to T12

D.
C8 to T1

A

The spinal cord is divided into four regions - from top to bottom, these are cervical, thoracic, lumbar, and sacral.

a. Incorrect The lumbar region of the spine contains nerves that connect to the thigh, hip, and leg.
b. Incorrect The sacral region contains nerves that innervate the foot and the posterior and lateral aspects of the leg.
c. Incorrect T2 through T12 innervate the trunk.
d. CORRECT T1 is the first segment of the thoracic spinal chord and contains nerves that innervate the forearm and C8 innervates the ring and little fingers.

The correct answer is: C8 to T1

97
Q

Your colleague plans to use the “dismantling strategy” to identify the components of systematic desensitization that contribute most to its effectiveness. Being familiar with previous research using this strategy, you predict that she will find which of the following to be the critical component?
Select one:

A.
reciprocal inhibition

B.
negative reinforcement

C.
classical extinction

D.
escape conditioning

A

Although Wolpe originally developed systematic desensitization as an application of reciprocal inhibition, research using the dismantling strategy has found that reciprocal inhibition is not the critical component.

a. Incorrect See explanation for response c.
b. Incorrect See explanation for response c.
c. CORRECT The dismantling strategy involves comparing the effects of the various components of an intervention by administering different components to different participants. The use of this strategy to identify the effective components of systematic desensitization has found that the critical component is extinction of the anxiety response through repeated exposure to the CS without the US.
d. Incorrect See explanation for response c.

The correct answer is: classical extinction

98
Q

The DSM-5 diagnosis of Tourette’s Disorder requires the presence of:
Select one:

A.
at least one vocal tic and one motor tic.

B.
at least one vocal tic and multiple motor tics.

C.
multiple vocal tics and at least one motor tic.

D.
multiple vocal and multiple motor tics.

A

Answer B is correct: The diagnosis of Tourette’s Disorder requires the presence of at least one vocal tic and multiple motor tics that have persisted for more than one year and had an onset before 18 years of age.

Answers A, C, and D: See explanation for answer B.

The correct answer is: at least one vocal tic and multiple motor tics.

99
Q

Richard Stuart’s operant interpersonal therapy combines the principles of operant conditioning with:
Select one:

A.
social role theory.

B.
social constructionism.

C.
object relations theory.

D.
social exchange theory.

A

Stuart describes successful marriages as involving a high degree of reciprocal positive behavioral exchanges.

a. Incorrect See explanation for response d.
b. Incorrect See explanation for response d.
c. Incorrect See explanation for response d.
d. CORRECT A primary influence on Stuart’s approach to behavioral marital therapy was social exchange theory, which conceptualizes satisfaction in relationships in terms of cost-reward ratios.

The correct answer is: social exchange theory.

100
Q

According to Hersey and Blanchard’s (1974) situational leadership model, a ________ leader is most effective for an employee who is low in ability and high in motivation.
Select one:

A.
telling

B.
selling

C.
delegating

D.
participating

A

Hersey and Blanchard’s situational leadership model distinguishes between the four leadership styles listed in the answers to this question and proposes that the best style is the one that matches the employee’s ability and motivation.

a. Incorrect A telling leader is best for employees who are low in both ability and motivation.
b. CORRECT A selling leader is best for employees who are low in ability but high in motivation.

Incorrect A delegating leader is best for employees who are high in both ability and motivation.

d. Incorrect A participating leader is best for employees who are high in ability but low in motivation.

The correct answer is: selling

101
Q

In adults, Major Depressive Disorder occurs about twice as often in females than in males. This gender difference:
Select one:

A.
is the opposite of what is found in prepubertal children.

B.
is the same as what is found in prepubertal children.

C.
does not become evident until adolescence.

D.
does not become evident until early adulthood.

A

Answer C is correct: Although there is some evidence that young girls exhibit more risk factors for depression during childhood, it is not until puberty that the gender difference in rates of Major Depressive Disorder becomes evident.

Answers A, B, and D: See explanation for answer C.

The correct answer is: does not become evident until adolescence.

102
Q

Common side effects of ________ include stomach cramps, nausea, vomiting, diarrhea, insomnia, nervousness, headache, and joint and muscle pain.
Select one:

A.
disulfiram (Antabuse)

B.
methylphenidate (Ritalin)

C.
naltrexone (ReVia)

D.
propranolol (Inderal)

A

For the exam, you want to be familiar with the side effects of the drugs listed in the answers to this question. This information is provided in the Physiological Psychology and Psychopharmacology chapter of the written study materials. Useful active study strategies for learning drug side effects are to create and frequently review a table of side effects and write and answer matching questions.

a. Incorrect See explanation for response c.
b. Incorrect See explanation for response c.
c. CORRECT The symptoms listed in this question are common side effects of naltrexone, which is an opioid receptor antagonist that blocks the craving for and reinforcing effects of alcohol.
d. Incorrect See explanation for response c.

The correct answer is: naltrexone (ReVia)

103
Q

REM sleep is ordinarily preceded by Stage 3 and Stage 4 sleep except in:
Select one:

A.
older adults.

B.
newborns.

C.
insomniacs and “long-sleepers.”

D.
newborns and older adults.

A

As noted in the Physiological Psychology and Psychopharmacology chapter in the written study materials, sleep patterns vary somewhat over the lifespan.

a. Incorrect See explanation for response b.
b. CORRECT In newborns, an EEG indicates only two distinct sleep patterns: REM and NREM (quiet) sleep. Also, during the first few months, REM precedes NREM (the sleep period begins with REM sleep).
c. Incorrect See explanation for response b.
d. Incorrect See explanation for response b.

The correct answer is: newborns.

104
Q

For Miller and Dollard (1941), the approach-approach conflict:
Select one:

A.
is a dilemma because as soon as the individual approaches one positive goal, the pull of the other positive goal increases in strength.

B.
is a dilemma that is often resolved by “leaving the field” and choosing a third goal.

C.
is not really a dilemma since, once the individual begins to approach one positive goal, the strength of the pull of the other positive goal decreases.

D.
is an unpredictable dilemma because it is impossible to ever know if the appropriate goal has been selected.

A

Miller and Dollard distinguish between three conflicts: approach-avoidance, avoidance-avoidance, and approach-approach.

a. Incorrect This is not a prediction made by Miller and Dollard but does sound somewhat like what Lewin concluded about this type of dilemma.
b. Incorrect This sounds more like what Lewin concluded about avoidance-avoidance conflicts.
c. CORRECT According to Miller and Dollard, the approach-approach conflict is not really a dilemma. When it does arouse conflict, it is because there are hidden or latent avoidance conditions operating.
d. Incorrect See explanations above.

The correct answer is: is not really a dilemma since, once the individual begins to approach one positive goal, the strength of the pull of the other positive goal decreases.

105
Q

On the MMPI-2, higher-than-normal L and K scale scores with a lower-than-normal F scale score has been linked to which of the following?
Select one:

A.
Malingering

B.
Schizophrenia

C.
somatization of psychological problems

D.
parental alientation syndrome

A

Knowing that the L, K, and F scales are validity scales may have helped you eliminate answers b and c.

a. Incorrect Malingering has been linked to several score patterns, including L and K scale scores around 50 with a slightly elevated F scale and a “saw-toothed” pattern on the clinical scales.
b. Incorrect Schizophrenia has been linked to the “psychotic valley” (scores on scales 6 and 8 substantially higher than the score on scale 7).
c. Incorrect Somatizationi of psychological problems has been linked to the “neurotic triad” (scores on scales 1 and 3 substantially higher than the score on scale 2).
d. CORRECT Siegel and Langsford (1998) compared the MMPI-2 profiles of mothers undergoing a custody evaluation as part of a divorce proceeding and found that both had a defensve pattern on the L, K, and F scales but that those mothers who also exhibited behaviors associated with parental alientation syndrome had even more extrems scores on these scales, with L and K being higher-than-normal and F being lower-than-normal.

The correct answer is: parental alientation syndrome

106
Q

A person with anosognosia will:
Select one:

A.
be unable to recognize parts of her own body.

B.
be unable to recognize familiar faces.

C.
deny the paralysis in her left leg.

D.
report an area of blindness in her left visual field.

A

Anosognosia is one of several types of agnosia. All agnosias are characterized by a failure of recognition that is NOT due to a sensory deficit or verbal or intellectual impairment.

a. Incorrect This is called asomatognosia and is due to damage to the somatosensory cortex. It usually involves the left side of the body.
b. Incorrect Prosopagnosia is the inability to recognize familiar faces.
c. CORRECT Anosognosia involves a failure to recognize one’s own neurological symptoms (e.g., paralysis). It often accompanies asomatognosia.
d. Incorrect This is called a scotoma and is due to damage to the primary visual cortex.

The correct answer is: deny the paralysis in her left leg.

107
Q

When making the appointment with a new client for her first therapy session, Dr. Steven Sue asks the client to notice whether any changes in her behavior occur between their phone conversation and her first therapy session. Then, during the first session, Dr. Sue asks the client what she noticed. Apparently, Dr. Sue is a practitioner of:
Select one:

A.
motivational interviewing.

B.
interpersonal therapy.

C.
solution-focused therapy.

D.
reality therapy.

A

For the exam, you want to be familiar with the major assumptions and processes of the four therapies listed in the answers to this question.

a. Incorrect See explanation for response c.
b. Incorrect See explanation for response c.
c. CORRECT Asking a client to notice if any changes in behavior occur between the initial phone conversation and first therapy session is one of the techniques used by solution-focused therapists to obtain information that can help identify solutions to the client’s problem.
d. Incorrect See explanation for response c.

The correct answer is: solution-focused therapy.

108
Q

As predicted by the Taylor-Russell tables, the incremental validity of a selection test that has a validity coefficient of .50 will be the greatest when:
Select one:

A.
the selection ratio is .05 and the base rate is .20.

B.
the selection ratio is .05 and the base rate is .50.

C.
the selection ratio is .95 and the base rate is .20.

D.
the selection ratio is .95 and the base rate is .50.

A

The Taylor-Russell Tables provide information on a test’s incremental validity for various combinations of base rates, selection ratios, and validity coefficients.

a. Incorrect See explanation for response b.
b. CORRECT A test with even a low or moderate validity coefficient can improve decision-making accuracy when the selection ratio is low (e.g., .05) and the base rate is moderate (near .50). Fir additional information on this topic, see the Industrial-Organizational Psychology chapter of the written study materials.
c. Incorrect See explanation for response b.
d. Incorrect See explanation for response b.

The correct answer is: the selection ratio is .05 and the base rate is .50.

109
Q

When a newly developed test consists of 100 true/false items, the ______________ can be used as an alternative to coefficient alpha for evaluating its internal consistency reliability.
Select one:

A.
Spearman-Brown formula

B.
coefficient of concordance

C.
Cohen’s kappa statistic

D.
Kuder-Richardson Formula 20

A

Answer D is correct. For the exam, you want to be familiar with all of the formulas listed in the answers to this question. These are described in the Test Construction chapter of the written study materials. Kuder-Richardson Formula 20 (KR-20) is an alternative to coefficient alpha when items are scored dichomotously (right or wrong), which would be the case for true/false items.

The correct answer is: Kuder-Richardson Formula 20

110
Q

Due to improved treatments for acute lymphacytic leukemia, at least 50% of children with this condition now have a normal life expectancy. Research investigating the cognitive impact of treatments for this disorder - that is, CNS chemotherapy and irradiation - has found that:
Select one:

A.
CNS chemotherapy and irradiation are both associated with neurocognitive deficits.

B.
CNS chemotherapy is associated with neurocognitive deficits but irradiation is not.

C.
CNS irradiation is associated with neurocognitive deficits but chemotherapy is not.

D.
any apparent negative impact of either treatment on neurocognitive functioning is actually due to the disease.

A

Answer A is correct: The research in this area is not entirely consistent, but it does appear that CNS chemotherapy and irradiation treatments for children with cancer do have some negative effects on their cognitive functioning, especially in terms of academic performance. [See, e.g., R. T. Brown and A. Madan-Sevain, Cognitive, neuropsychological, and academic sequelae in children with leukemia, Journal of Learning Disabilities, 26(2), 74-90, 1993.]

Answers B, C, and D: See explanation for answer A.

The correct answer is: CNS chemotherapy and irradiation are both associated with neurocognitive deficits.

111
Q

__________ drugs are sometimes used to alleviate the extrapyramidal symptoms produced by an antipsychotic drug but are themselves associated with a number of undesirable side effects including dry mouth, blurred vision, urinary retention, constipation, and tachycardia.
Select one:

A.
Anticholinergic

B.
Adrenergic

C.
Cholinergic

D.
Antiserotonergic

A

Being familiar with the anticholinergic side effects of antihistamines, antidepressants, antipsychotics, and a number of other drugs would have helped you identify the correct answer to this question.

a. CORRECT The common side effects of the anticholinergic drugs include those listed in the question. These drugs exert their effects by antagonizing the action of acetylcholine at muscarnic receptors.
b. Incorrect See explanation for response a.
c. Incorrect See explanation for response a.
d. Incorrect See explanation for response a.

The correct answer is: Anticholinergic

112
Q

Although most health care workers recognize the importance of hand washing for reducing hospital-based infection, frequency of hand washing by hospital personnel is low. Studies evaluating the effectiveness of various strategies for increasing hand washing by health care workers suggest that the most effective approach combines:
Select one:

A.
rewards, sanctions, and provision of moisturized soaps.

B.
automated sinks and provision of moisturized soaps.

C.
continuous performance feedback with appropriate rewards and sanctions.

D.
education, reminders, and continuous performance feedback.

A

This is a difficult question since the research on methods for improving hand hygiene (hand washing) among health care workers has not produced entirely consistent results.

a. Incorrect Rewards and punishments have not been found to be particularly effective. In one study, 79% of the health care workers surveyed reported that rewards and punishments would not improve hand washing [A. D. Harris et al., A survey on handwashing practices and opinions of healthcare workers, Journal of Hospital Infections, 45(4), 318-321, 2000].
b. Incorrect The research suggests that, while automated sinks may improve the quality of hand washing, they actually reduce the frequency of hand washing. There is also evidence that providing moisturized soap has little or no effect on hand-washing frequency.
c. Incorrect See explanation for responses a and d.
d. CORRECT Although the research on this issue has not provided consistent results, overall, it has shown that a multi-component approach that incorporates education is most effective. For example, based on their critical review of the literature, S. Naikoba and A. Hayward conclude that “multifaceted approaches which combine education with written materials, reminders, and continued feedback of performance can have an important effect on handwashing compliance and rates of hospital-acquired infection” (The effectiveness of interventions aimed at increasing handwashing in healthcare workers - a systematic review, Journal of Hospital Infection, 47, 173-180, 2001).

The correct answer is: education, reminders, and continuous performance feedback.

113
Q

__________ is the most frequent cause of the ethical complaints brought against psychotherapy supervisors.
Select one:

A.
Sexual harassment

B.
Breach of confidentiality

C.
Lack of timely feedback

D.
Incompetence

A

There is very little empirical research on supervision, and many of the conclusions drawn in the literature seem to be based on anecdotal evidence.

a. Incorrect See explanation for response c.
b. Incorrect See explanation for response c.
c. CORRECT Since the primary purpose of supervision is to provide supervisees with feedback about their performance, it makes sense that this would be the most frequently cited problem. See, e.g., N. Ladany, D. Lehrman-Waterman, M. Molinaro, & B. Wolgast, Psychotherapy supervisor ethical practices: Adherence to guidelines, the supervisory working alliance, and supervisee satisfaction, The Counseling Psychologist, 27(3), 443-475, 1999.
d. Incorrect See explanation for response c.

The correct answer is: Lack of timely feedback

114
Q

Which of the following imaging techniques would be used to obtain a “metabolic map” of the brain during various mental and physical activities?
Select one:

A.
CT

B.
PET

C.
MRI

D.
NMR

A

In this situation, it is the function of the brain that is of interest.

a. Incorrect CT is a structural imaging technique.
b. CORRECT PET scans provide information on the functions of the brain (e.g., metabolic functions).
c. Incorrect MRI is also a structural technique.
d. Incorrect NMR (nuclear magnetic resonance) is another name for MRI, which is a structural imaging technique. (An fMRI is a functional technique.)

The correct answer is: PET

115
Q

Which of the following are common symptoms of both Broca’s and Wernicke’s aphasia?
Select one:

A.
anomia and problems with repetition

B.
anomia and paraphasia

C.
paraphasia and problems with repetition

D.
paraphasia, anomia, and problems with repetition

A

For the exam, you want to be familiar with the language impairments associated with the various types of aphasia. These are described in the Physiological Psychology and Psychopharmacology chapter of the written study materials.

a. CORRECT Broca’s and Wernicke’s aphasia both involve anomia (an inability to name a familiar object, attribute, or action) and problems with repetition. However, paraphasia is a common symptom of Wernicke’s aphasia only.
b. Incorrect See explanation for response a.
c. Incorrect See explanation for response a.
d. Incorrect See explanation for response a.

The correct answer is: anomia and problems with repetition

116
Q

A Type I error occurs when a researcher:
Select one:

A.
rejects a true null hypothesis.

B.
retains a true null hypothesis.

C.
rejects a false null hypothesis.

D.
retains a false null hypothesis.

A

Being familiar with the decision outcome table included in the Statistics and Research Design chapter of the written study materials may have helped you identify the correct answer to this question.

a. CORRECT A Type I error occurs when a researcher concludes that the independent variable has had an effect on the dependent variable but the apparent effect is actually due to sampling error – i.e., the researcher has rejected a true null hypothesis.
b. Incorrect This is a correct decision.
c. Incorrect This is a correct decision.
d. Incorrect A Type II error occurs when a researcher retains a false null hypothesis.

The correct answer is: rejects a true null hypothesis.

117
Q

Frame-of-reference (FOR) training is used to:
Select one:

A.
improve the accuracy of job descriptions.

B.
maximize transfer of training.

C.
reduce rater idiosyncrasies.

D.
improve the face validity of criterion measures.

A

An effective way to improve rater accuracy is to provide raters with adequate training.

a. Incorrect See explanation for response c.
b. Incorrect See explanation for response c.
c. CORRECT Frame-of-reference training improves rater accuracy by providing raters with a common frame-of-reference for evaluating job performance.
d. Incorrect See explanation for resonse c.

The correct answer is: reduce rater idiosyncrasies.

118
Q

According to Dawis and Lofquist’s (1984) theory of work adjustment, a worker’s satisfactoriness depends on the degree to which the worker’s:
Select one:

A.
abilities match the ability requirements of the job.

B.
needs match the reinforcers provided by the job.

C.
self-image matches the status provided by the job.

D.
basic interests match the characteristics of the work environment.

A

The theory of work adjustment is a person-environment fit theory of career development and links tenure and other job outcomes to two factors – satisfaction and satisfactoriness.

a. CORRECT Satisfactoriness depends on the extent to which the worker’s skills correspond to the skill demands of the job.

. b. Incorrect Satisfaction depends on the extent to which the worker’s needs correspond to reinforcers provided by the job.

c. Incorrect See explanation above.
d. Incorrect See explanation above.

The correct answer is: abilities match the ability requirements of the job.

119
Q

Baltes et al.’s (1999) meta-analysis of research on the compressed (four-day) workweek found that it results in:
Select one:

A.
an increase in job satisfaction but a decrease in supervisor ratings of performance.

B.
increases in job satisfaction and supervisor ratings of performance.

C.
increases in objective measures of productivity and organizational commitment.

D.
a decrease in objective measures of productivity and an increase in absenteeism.

A

Studies on the compressed workweek are few and have, unfortunately, not produced entirely consistent results. However, this question is asking specifically about the results of the Baltes et al. (1999) meta-analysis, which are summarized in the Industrial-Organizational Psychology chapter of the written study materials.

a. Incorrect See explanation for response b.
b. CORRECT Baltes and colleagues found that the compressed workweek had positive effects on supervisor ratings of employee performance, employee overall job satisfaction, and employee satisfaction with the work schedule, with the effects being strongest for employee attitudes.
c. Incorrect See explanation for response b.
d. Incorrect Baltes et al. found that the compressed workweek had little effects on objective measures of job productivity or absenteeism.

The correct answer is: increases in job satisfaction and supervisor ratings of performance.

120
Q

Which of the following theories proposes that the development of gender-role identity involves recognizing what males and females are “supposed to do” in one’s own sociocultural environment and then adapting one’s behaviors and attitudes to fit those expectations?
Select one:

A.
Freuds psychodynamic theory

B.
Mischels social learning theory

C.
Bems gender schema theory

D.
Kohlbergs cognitive-developmental theory

A

The four theories listed in the answers are the primary explanations for the development of gender-role identity.

a. Incorrect Freud’s theory of gender-role identity focuses on resolution of the psychosexual crisis of the phallic stage of development.
b. Incorrect Although Mischel’s social learning theory does emphasize the role of social factors, it focuses primarily on the impact of punishment and reinforcement on the development of gender-role identity.
c. CORRECT Bem’s gender schema theory is classified as a social-cognitive approach because it emphasizes both social (especially sociocultural) factors and cognitive processes. According to this theory, children develop schemas about what is expected of them as girls or boys and then apply those schemas to their own behavior.
d. Incorrect Kohlberg’s cognitive-developmental theory focuses on the cognitive processes that underlie the development of a gender-role identity.

The correct answer is: Bems gender schema theory

121
Q

You have been seeing a couple in conjoint therapy for seven months and anticipate that treatment will be ending soon. One day you receive a call from the wife who says she would like to start seeing you in individual therapy. Your best course of action would be to:
Select one:

A.
make an appointment with her as long as you believe that she will benefit from individual therapy.

B.
make an appointment with her only if it is your practice to see members of couples conjointly and individually.

C.
suggest that you discuss this with her during the next conjoint session.

D.
tell her that you’d be glad to see her individually once conjoint treatment has been terminated.
Feedback

A

This is a difficult question because not enough information is given to know why the request for individual therapy was made and what has already been discussed about individual therapy with the couple.

a. Incorrect See explanation for response c.
b. Incorrect See explanation for response c.
c. CORRECT To identify why the woman is seeking individual therapy and to avoid “secrets” and conflicts regarding confidentiality, the best course of action would be to discuss the possibility of seeing the woman in individual therapy during the next conjoint session.
d. Incorrect See explanation for response c.

The correct answer is: suggest that you discuss this with her during the next conjoint session.

122
Q

In organizations, the level of affective commitment is likely to be LEAST predictive of which of the following?
Select one:

A.
job satisfaction

B.
productivity

C.
job motivation

D.
turnover

A

As noted in the Industrial-Organizational Psychology chapter of the written study materials, two types of organizational commitment are distinguished - continuance commitment and affective commitment. Of these, affective commitment is most predictive of work-related outcomes.

a. Incorrect See explanation for response b.
b. CORRECT Of the outcomes listed in the answers, affective commitment is least predictive of productivity.
c. Incorrect See explanation for response b.
d. Incorrect See explanation for response b.

The correct answer is: productivity

123
Q

Learning a sequence of operant responses that each act as a secondary reinforcer and that eventually end in a primary reinforcer is known as:
Select one:

A.
shaping.

B.
chaining.

C.
differential reinforcement.

D.
classical conditioning.

A

To answer this question, you must recognize that the procedure described is a type of operant conditioning (the term “reinforcer” is a clue) and be familiar with the various operant conditioning techniques. If you are unfamiliar with chaining (the correct answer), perhaps you could have chosen it as the correct answer though a process of elimination.

a. Incorrect This is not an accurate description of shaping, which involves reinforcing successive approximations to the desired behavior.
b. CORRECT Chaining, which is believed to account for the acquisition of complex behaviors, involves the association of responses such that each response acts as a secondary reinforcer and as a discriminative stimulus for the following response. The final reinforcer (the reinforcer that is delivered at the end of the “chain”) is a primary reinforcer.
c. Incorrect The information in the question does not describe differential reinforcement, in which the target behavior is decreased by reinforcing behaviors other than the target behavior.
d. Incorrect Classical conditioning occurs when a neutral stimulus is repeatedly paired with an unconditioned stimulus so that the neutral (conditioned) stimulus eventually elicits a response similar to the one elicited by the unconditioned stimulus. Although classical conditioning is involved in chaining (i.e., in the association between each discriminative stimulus and the following reinforcer), this is not the best answer of those given.

The correct answer is: chaining.

124
Q

A psychotherapist who has adopted an emic perspective believes that:
Select one:

A.
a particular mental disorder is manifested in similar ways in all cultures.

B.
the manifestation of a particular mental disorder is impacted by cultural values, norms, and attitudes.

C.
the primary goal of therapy should be altering the client’s internal state.

D.
the primary goal of therapy should be altering the client’s environment.

A

For the exam, you want to be familiar with all of the terms described in the answers to this question.

a. Incorrect Viewing a mental disorder as being manifested similarly in all cultures reflects an etic perspective.
b. CORRECT Viewing a mental disorder as being affected by cultural values, norms, and attitudes reflects an emic perspective.
c. Incorrect The goal of an autoplastic intervention is to change the individual so that he/she is better able to function effectively in his/her environment.
d. Incorrect The goal of an alloplastic interventin is to change the environment so that it better accommodates the individual.

The correct answer is: the manifestation of a particular mental disorder is impacted by cultural values, norms, and attitudes.

125
Q

When excellent performance on one measure can offset poor performance on another measure, which of the following techniques would be most useful for combining test scores?
Select one:

A.
multiple regression

B.
multiple cutoff

C.
multiple hurdle

D.
multiple baseline

A

This question describes a situation in which a compensatory technique would be the preferred technique for combining test scores.

a. CORRECT Multiple regression is a compensatory technique for combining test scores since a low score on one test can be offset (compensated for) by high scores on other tests.
b. Incorrect Multiple cutoff is a noncompensatory technique.
c. Incorrect Multiple hurdle is also a noncompensatory technique.
d. Incorrect “Multiple baseline” refers to a research design, not a method for combining test scores.

The correct answer is: multiple regression

126
Q

If an infant is unable to focus or coordinate one eye with the other and this problem is not corrected before age one, the child will always have problems with binocular vision. This best illustrates the concept of:
Select one:

A.
a sensitive period.

B.
a critical period.

C.
canalization.

D.
range of reaction.

A

Some aspects of development have critical periods, while others have sensitive periods.

a. Incorrect A sensitive period is longer in duration and more flexible than a critical period. Language, for example, is believed to have a sensitive period: Once the optimal period for being exposed to language has passed, the individual may still be able to develop language skills (although it may be more difficult to do so).
b. CORRECT The development of binocular vision is viewed by experts as having a critical period. Specifically, if an infant is not able to coordinate both eyes by age one, he/she will always have problems with binocular vision; and, if the problem is not corrected by age 5, binocular vision will never develop.
c. Incorrect Canalization is the tendency for genetic endowment to restrict the development of certain characteristics to one or a few outcomes.
d. Incorrect Range of reaction refers to the fact that each individual responds to the environment in a unique way because of his/her genetic endowment.

The correct answer is: a critical period.

127
Q

A psychologist wants to know if a husband’s participation in housework is related to the wife’s occupational status and the couple’s socioeconomic status. He obtains a sample of married men and determines the current occupational status of each man’s wife (working full-time outside the home, working part-time outside the home, working full- or part-time in the home, or not working) and their SES (upper, middle, or lower) and asks each man to estimate the average number of hours he spends doing housework each week. The appropriate test for analyzing the data the researcher collects in study is the:
Select one:

A.
one-way ANOVA.

B.
two-way ANOVA.

C.
MANOVA.

D.
multiple-sample chi-square.

A

The first step in determining the correct answer to this question is to identify the study’s independent and dependent variables by translating the information provided into the following question: What are the effects of a wife’s occupational status and the couple’s socioeconomic status on the number of hours the husband spends doing housework? This question indicates that the study has two independent variables (occupational status and socioeconomic status) and one dependent variable (hours doing housework) and that the dependent variable is measured on a ratio scale.

a. Incorrect See explanation for response b.
b. CORRECT The two-way ANOVA is the appropriate statistical test for a study that has two independent variables and one dependent variable that is measured on a ratio scale.
c. Incorrect See explanation for response b.
d. Incorrect See explanation for response b.

The correct answer is: two-way ANOVA.

128
Q

When asked about their abusive activities, child sex abusers are most likely to:
Select one:

A.
acknowledge what they did was wrong but say they werent able to control their impulses.

B.
initially deny the abuse but eventually admit their actions and express remorse.

C.
admit their actions but minimize the harmful effects of those actions on the child.

D.
admit their actions, express shame and embarrassment, and say they will never do it again.

A

The research on child sex abusers has found that they share a number of characteristics.

a. Incorrect See explanation for response c.
b. Incorrect See explanation for response c.
c. CORRECT R. Underwager and H. Wakefield report the results of a study of 94 child sex abusers [Sex offender treatment requiring admission of guilt, presented at the 15th Annual Symposium of the American College of Forensic Psychology, Santa Fe, New Mexico, 1999]. The largest proportion of their sample (57%) admitted engaging in sexual behaviors with a child but minimized the behavior in some way - e.g., by denying that the child was harmed or by blaming the child. Their results are consistent with many other studies indicating that the majority of child sex abusers and other sex offenders minimize, rationalize, or justify their abusive behaviors.
d. Incorrect See explanation for response c.

The correct answer is: admit their actions but minimize the harmful effects of those actions on the child.

129
Q

Dr. Hyer, a psychologist who heads the personnel committee at a mental health facility, recommends that a psychologist who has an unresolved charge of sexual harassment against him not be considered for a promotion. In terms of ethical guidelines published by the American and Canadian Psychological Associations, Dr. Hyer:
Select one:

A.
has acted ethically since sexual harassment is explicitly prohibited by ethical guidelines.

B.
acted ethically as long as he is willing to consider the psychologist for promotion if he is acquitted of the harassment charge.

C.
has acted unethically by violating the ethical requirement that psychologists not deny promotions on the basis of pending sexual harassment charges.

D.
has acted unprofessionally but has not violated ethical guidelines.

A

This is one of the issues that is explicitly addressed by the Ethics Code.

a. Incorrect See explanation for response c.
b. Incorrect See explanation for response c.
c. CORRECT Standard 1.08 of the APA’s Ethics Code applies in this situation. It specifically prohibits denying a person employment, promotion, etc. “based solely upon their having made or their being the subject of an ethics complaint.” This prohibition does not necessarily, however, preclude such actions when a charge against an individual has been proven. This answer is also most consistent with Principle I.13 of the Canadian Code of Ethics for Psychologists.
d. Incorrect See explanation for response c.

The correct answer is: has acted unethically by violating the ethical requirement that psychologists not deny promotions on the basis of pending sexual harassment charges.

130
Q

According to the ________ bootstrapping hypothesis, children use their knowledge of the meaning of words to figure out the structure of language.
Select one:

A.
syntactic

B.
phonological

C.
semantic

D.
morphological

A

Bootstrapping is used to describe language development in young children.

a. Incorrect See explanation for response c.
b. Incorrect See explanation for response c.
c. CORRECT The semantic bootstrapping hypothesis predicts that children use their understanding of the content of language to “bootstrap” the syntactical (grammatical) structure of language. Additional information on bootstrapping is provided in the Lifespan Development chapter of the written study materials.
d. Incorrect See explanation for response c.

The correct answer is: semantic

131
Q

When determining a treatment plan for mild to moderate hypertension, it is important to keep in mind that:
Select one:

A.
EMG feedback is the most effective treatment.

B.
EKG feedback plus medication is the most effective treatment.

C.
biofeedback is ineffective for hypertension.

D.
blood pressure feedback is an effective training in general relaxation.

A

This is a difficult question since the research has been inconsistent (see, e.g., I. S. Goldstein, Biofeedback in the treatment of hypertension, in L. White and B. Tursky, Clinical Biofeedback: Efficiency and Mechanisms, New York: Guilford Press, 1982). The BEST answer to this type of question is usually the one that does not claim any one treatment to be superior to all others.

a. Incorrect Although EMG feedback has been found useful, it hasn’t been found to be the most effective treatment for hypertension.
b. Incorrect An EKG provides information about cardiac functioning and is not useful as a treatment for hypertension.
c. Incorrect While the degree of its effectiveness has not been established, biofeedback (especially blood pressure feedback) has been found useful as a treatment for mild to moderate hypertension.
d. CORRECT This response is most consistent with the conclusion that can be drawn from the research. While blood pressure feedback is effective, many studies have found that training in general relaxation is equally effective for mild to moderate cases.

The correct answer is: blood pressure feedback is an effective training in general relaxation.

132
Q

Feature-integration theory (Treisman & Gelade, 1980) would be of most interest to a reasearcher who is investigating:
Select one:

A.
focused auditory attention.

B.
focused visual attention.

C.
divided attention.

D.
selective attention.

A

Feature-integration theory is used to explain how we perceive objects as complete entities rather than as meaningless collections of features (e.g., color, texture, size).

a. Incorrect Feature-integration theory is a theory of focused visual attention.
b. CORRECT According to Treisman and Gelade (1980), focused visual attention is necessary to perceive objects as objects rather than as a collection of unrelated features. Additional information about feature-integration theory is provided in the Learning Theory chapter of the written study materials.
c. Incorrect See explanation above.
d. Incorrect See explanation above.

The correct answer is: focused visual attention.

133
Q

A personnel director decides to raise a selection test’s cutoff score when using the test to assist in hiring decisions. Most likely the personnel director has made this decision in order to:
Select one:

A.
increase the number of true positives.

B.
decrease the number of false positives.

C.
increase the number of false negatives.

D.
decrease the number of true negatives.

A

Raising the predictor cutoff score decreases the number of true and false positives and increases the number of true and false negatives.

a. Incorrect Raising the predictor cutoff score would decrease the number of true positives.
b. CORRECT Most likely, a personnel director would raise the predictor cutoff to decrease the number of false positives since these individuals “cost the company money” (i.e., they would be hired on the basis of their test scores but would be unsuccessful on the job).
c. Incorrect Although raising the predictor cutoff score increases the number of false negatives, this would not be considered desirable in most situations - i.e., a personnel director would not want to increase the number of people who are rejected (not hired) on the basis of their selection test scores but would have been successful on the job if they had been hired.
d. Incorrect Raising the predictor cutoff increases the number of true negatives.

The correct answer is: decrease the number of false positives.

134
Q

Among men, the lowest rates of suicide are for those who are:
Select one:

A.
single.

B.
divorced.

C.
married.

D.
cohabiting.

A

Answer C is correct: Studies have found that divorced and separated individuals have the highest risk for suicide and are about twice as likely to commit suicide than those who are married. Conversely, those who are married are least likely to commit suicide. Note that there is some evidence that the relationship between marital status and suicide risk may be statistically significant for men only.

Answers A, B, and D: See explanation for answer C.

The correct answer is: married.

135
Q

Gerstmann’s Syndrome is characterized by:
Select one:

A.
dysgraphia, finger agnosia, acalculia, and right-left disorientation.

B.
responding to simple questions with absurd, approximate, or inappropriate answers.

C.
strong oral tendencies, hypersexuality, lack of concentration, and alterations in appetite.

D.
anterograde amnesia, confusion, and confabulation.

A

Gerstmann’s Syndrome is caused by damage to left (dominant) parietal lobe.

a. CORRECT These are the four primary symptoms of Gerstmann’s Syndrome.
b. Incorrect These symptoms are characteristic of Ganser Syndrome, which is a rare disorder most often seen in patients under examination for commitment and prisoners awaiting trial.
c. Incorrect These are characteristic symptoms of Kluver-Bucy Syndrome.
d. Incorrect These symptoms are observed in Korsakoff’s Syndrome.

The correct answer is: dysgraphia, finger agnosia, acalculia, and right-left disorientation.

136
Q

For a DSM-5 diagnosis of Schizophrenia or Schizophreniform Disorder, an individual must exhibit at least two characteristic symptoms during a one-month period, one of which must be:
Select one:

A.
derealization or depersonalization.

B.
disorganized speech or disorganized behavior.

C.
delusions, hallucinations, or a negative symptom.

D.
delusions, hallucinations, or disorganized speech.

A

A diagnosis of Schizophrenia requires the presence of at least two characteristic symptoms during a one-month period with at least one symptom being delusions, hallucinations, or disorganized speech plus continuous signs of the disturbance for at least six months. A diagnosis of Schizophreniform Disorder has the same requirements except that the duration of the disturbance must be for at least one month but less than six months.

Answers A, B, and C: See explanation for answer D.

The correct answer is: delusions, hallucinations, or disorganized speech.

137
Q

An ipsative score provides information about:
Select one:

A.
an examinee’s score on a test relative to scores obtained by examinees in the same age group.

B.
an examinee’s score on a test relative to scores obtained by examinees in the same academic grade.

C.
an examinee’s score on one scale of a test relative to his/her scores on the other scales.

D.
an examinee’s score on a test relative to a predefined standard of performance.

A

An ipsative score is different from most other types of scores which either provide information on the examinee’s standing in a norm group or standing in terms of an absolute standard of performance.

a. Incorrect See explanation for response c.
b. Incorrect See explanation for response c.
c. CORRECT An ipsative score indicates the relative strength of an examinee’s status or performance on the various scales of the test.
d. Incorrect See explanation for response c.

The correct answer is: an examinee’s score on one scale of a test relative to his/her scores on the other scales.

138
Q

The __________ is best described as the “gateway to memory” because of its involvement in the storage of new information.
Select one:

A.
hippocampus

B.
hypothalamus

C.
thalamus

D.
R.A.S.

A

For the exam, you want to be familiar with the major functions of the brain structures listed in the answers to this question.

a. CORRECT Damage to the hippocampus is associated with memory impairments, especially impairments in retaining recently acquired information.
b. Incorrect The hypothalamus is involved in a number of important functions including the maintenance of the body’s internal balance (homeostasis).
c. Incorrect The thalamus relays sensory messages to the cortex.
d. Incorrect The reticular activating system (RAS) is involved in arousing the cortex and screening incoming information.

The correct answer is: hippocampus

139
Q

According to Sternberg (1999), traditional intelligence tests focus primarily on __________, which is one of the three abilities identified in his triarchic theory of intelligence.
Select one:

A.
analytical ability

B.
logical-mathematical ability

C.
verbal memory

D.
processing speed

A

Being familiar with the three intellectual abilities identified by Sternberg would have allowed you to identify the correct answer to this question.

a. CORRECT Sternberg’s three abilities are analytical, creative, and practical.
b. Incorrect See explanation above.
c. Incorrect See explanation above.
d. Incorrect See explanation above.

The correct answer is: analytical ability

140
Q

In principal components analysis, an eigenvalue is:
Select one:

A.
a lower-limit estimate of a test’s reliability.

B.
the correlation between a test and a component.

C.
the total variability in a test that is explained by the identified components.

D.
the total variability explained by an orthogonal component.

A

Principal components analysis is used to identify a set of variables (components) that explains the total variability in a set of test scores.

a. Incorrect See explanation for response d.
b. Incorrect This describes a factor loading.
c. Incorrect This describes a communality.
d. CORRECT In principal components analysis, each component has an eigenvalue, which indicates the total amount of variability explained by that component. For additional information on principal components analysis, see the Appendix in the Test Construction chapter of the written study materials.

The correct answer is: the total variability explained by an orthogonal component.

141
Q

For the assessment of Huntington’s disease, magnetic resonance imaging (MRI) is:
Select one:

A.
not useful because it is not sufficiently sensitive to detect the brain anomalies associated with this disorder.

B.
not useful because it provides information only on brain morphology, which is not helpful for assessing this disorder.

C.
useful for distinguishing between active and older inactive plaques in the periventricular regions.

D.
useful for detecting volume reduction in the basal ganglia, even in some asymptomatic individuals.

A

Knowing that Huntington’s disease is caused by abnormalities in the basal ganglia would have helped you identify the correct answer to this question.

a. Incorrect An advantage of MRI is that it’s very sensitive to anomalies and has better resolution than a CT scan.
b. Incorrect As noted below, information about structure is useful in the diagnosis of Huntington’s disease.
c. Incorrect Plaques in the periventricular regions are characteristic of multiple sclerosis, not Huntington’s disease.
d. CORRECT Structural and functional brain imaging techniques have both been used to assess brain pathology related to Huntington’s disease. There is evidence that MRI (a structural technique) can identify reduced volume in the basal ganglia among genetically at-risk individuals who have not yet exhibited overt symptoms of the disorder.

The correct answer is: useful for detecting volume reduction in the basal ganglia, even in some asymptomatic individuals.

142
Q

A researcher designs a study to test Vygotsky’s theory of cognitive development. Assuming that Vygotsky is right, the researcher will find that cognitive development is most affected by:
Select one:

A.
heredity.

B.
culture.

C.
external reinforcement.

D.
intrinsic motivation.

A

You may have been able to pick the correct response to this question by recalling that Vygotsky proposes that cognitive development is facilitated by social interactions that occur within the child’s zone of proximal development.

a. Incorrect See explanation for response b.
b. CORRECT Vygotsky believed that cognitive development is fostered by interpersonal interactions and, as a consequence, is affected by culture and other social factors. See the Lifespan Development chapter for additional information on Vygotsky’s theory.
c. Incorrect See explanation for response b.
d. Incorrect See explanation for response b.

The correct answer is: culture.

143
Q

In a research study, each participant was given a vague description of themselves that was supposedly based on an analysis of his/her handwriting. In fact, all participants received the same randomly derived description. When asked about the accuracy of the description, the majority of participants agreed that it was “highly accurate.” These results support the predictions of which of the following?
Select one:

A.
self-serving bias

B.
looking-glass self

C.
Barnum effect

D.
Zeigarnik effect

A

A study similar to the one described in this question was conducted by Dickson and Kelly (1985).

a. Incorrect The self-serving bias is the tendency to take credit for our successes but blame external events for our failures.
b. Incorrect The looking-glass self predicts that people learn about themselves by imagining how others perceive them.
c. CORRECT The Barnum effect predicts that people tend to accept vague or general descriptions of themselves as accurate descriptions. It has been used to explain why people believe that astrological predictions or descriptions are perceived as accurate by many people.
d. Incorrect The Zeigarnik effect refers to the tendency to recall goals or tasks that have not been completed more readily (often automatically and in an intrusive way) than those that have been completed.

The correct answer is: Barnum effect

144
Q

Because of its potentially damaging effects on therapy outcome, an important issue in therapy is premature termination. Studies examining client and therapist factors related to therapy dropout have indicated all of the following EXCEPT:
Select one:

A.
a client’s socioeconomic status is correlated with length of stay in treatment.

B.
the greater the therapist’s experience, the lower the premature dropout rate.

C.
there is no relationship between a client’s gender and the probability of premature termination.

D.
a client with high initial anxiety is more likely to drop out of therapy prematurely.

A

Although most research on therapy outcome has yielded inconclusive results, a few factors have been consistently linked to premature termination. Note that this question is asking about the factor that has NOT been linked with premature dropout from therapy.

a. Incorrect This is true. A number of studies have found premature termination rates to be higher among members of the lower socioeconomic classes.
b. Incorrect This is also true.
c. Incorrect This is true.
d. CORRECT This is the opposite of what is true: Clients with high levels of initial anxiety are most likely to stay in therapy.

The correct answer is: a client with high initial anxiety is more likely to drop out of therapy prematurely.

145
Q

Research on the effects of gender on goal-setting indicates that:
Select one:

A.
goal-setting increases the performance of both males and females when goals are participatively set or assigned.

B.
goal-setting increases the performance of both males and females only when goals are participatively set.

C.
goal-setting increases the performance of males but not females when goals are assigned but increases the performance of males and females when goals are participatively set.

D.
goal-setting increases the performance of females but not males when goals are assigned but increases the performance of males and females when goals are participatively set.

A

The effects of gender on goal-setting are discussed by E. A. Locke and G. P. Latham (A Theory of Goal-Setting and Task Performance, Englewood Cliffs, NJ: Prentice-Hall, 1990).

a. CORRECT The few studies that have examined the effects of gender have found that, overall, goal-setting is equally effective for males and females.
b. Incorrect See explanation for response a.
c. Incorrect See explanation for response a.
d. Incorrect See explanation for response a.

The correct answer is: goal-setting increases the performance of both males and females when goals are participatively set or assigned.

146
Q

Renee R., age 16, is brought to therapy by her mother who says the girl has recently become “another person.” She says her daughter used to be easy-going and was well-liked at school by her peers and teachers. However, for about the last three weeks, Renee has been constantly irritable and “on the go,” has been smoking and drinking, and has started having trouble at school. When the therapist interviews Renee, he learns that she is engaging in high-risk sexual behavior but feels that nothing bad can happen to her. Based on these symptoms, the most likely diagnosis is:
Select one:

A.
ADHD.

B.
Cyclothymic Disorder.

C.
Bipolar I Disorder.

D.
Bipolar II Disorder.

A

Answer C is correct: The sudden change in Renee’s behavior and the nature and duration of her symptoms are most suggestive of a manic episode. Bipolar I Disorder requires the presence of one or more manic episodes.

Answer A: Although some of the girl’s symptoms are consistent with ADHD in adolescence, a diagnosis of this disorder requires an onset of symptoms prior to age 12.

Answer B: Cyclothymic Disorder requires fluctuating hypomanic and depressive symptoms for at least one year in children and adolescents.

Answer D: Bipolar II disorder is characterized by a combination of major depressive and hypomanic episodes.

The correct answer is: Bipolar I Disorder.

147
Q

Group polarization has occurred when:
Select one:

A.
group members are split in their solution to a problem.

B.
group members make riskier decisions as a group than they would have as individuals.

C.
group members make either riskier or more conservative decisions as a group than they would have as individuals.

D.
group members are encouraged to think alike and all dissent is discouraged.

A

Group polarization refers to the tendency for a group’s decision to be more extreme (polarized) that the decisions that individual members would have made alone.

a. Incorrect This is not an accurate description of group polarization.
b. Incorrect This describes the “risky shift” phenomenon.
c. CORRECT This is an accurate description of group polarization.
d. Incorrect This sounds more like groupthink.

The correct answer is: group members make either riskier or more conservative decisions as a group than they would have as individuals.

148
Q
Michael Rutter (1979) identified several factors ("Rutter's indicators") that increase the risk for psychopathology in children and adolescents. These include all of the following except:
Select one:

A.
low socioeconomic status.

B.
large family size and overcrowding.

C.
maternal psychopathology.

D.
racial/ethnic minority status.

A

Rutter’s (1979) indicators for psychopathology are low SES, marital discord, large family size and overcrowding, paternal criminality, maternal psychiatric disorder, and being placed in foster care.

a. Incorrect Low SES was identified by Rutter as a high-risk factor.
b. Incorrect Rutter also identified large family size as a high-risk factor.
c. Incorrect Maternal psychopathology was also identified as a risk factor by Rutter.
d. CORRECT Racial/ethnic minority status is not one of Rutter’s indicators.

The correct answer is: racial/ethnic minority status.

149
Q

Select one:

A.
increasing the availability of dopamine.

B.
decreasing the availability of serotonin.

C.
increasing sensitivity to epinephrine.

D.
decreasing sensitivity to norepinephrine.

A

Cocaine exerts its effects primarily on neurons in the limbic system and certain areas of the midbrain.

a. CORRECT It appears that cocaine blocks the reuptake of dopamine, thereby allowing it to accumulate in the synapses and continue to stimulate receiving cells.
b. Incorrect See explanation above.
c. Incorrect See explanation above.
d. Incorrect See explanation above.

The correct answer is: increasing the availability of dopamine.

150
Q

In a scatterplot, the regression lines for a test for two different groups of examinees differ substantially in terms of slope. This suggests that the test has:
Select one:

A.
a lack of factorial validity.

B.
a lack of convergent validity..

C.
divergent validity

D.
differential validity.

A

The slope of a regression line for a test is directly related to the test’s criterion-related validity: The steeper the slope, the greater the validity.

a. Incorrect Factorial validity refers to the degree to which a test or test item correlates with factors that it would be expected to correlate with in a factor analysis.
b. Incorrect Convergent validity refers to the degree to which a test correlates with measures of the same or a similar construct.
c. Incorrect Divergent validity refers to the degree to which a test does not correlate with measures of an unrelated construct.
d. CORRECT A test has differential validity when it has different validity coefficients for different groups, which is what is suggested by different regression line slopes in a scatterplot.

The correct answer is: differential validity.

151
Q

Dr. Jones, a licensed clinical psychologist, hires a psychology graduate student to administer the MMPI-2 to some of his clients. This is:
Select one:

A.
unethical since psychological tests should be administered only by licensed professionals.

B.
ethical as long as Dr. Jones accepts full responsibility for the consequences of testing.

C.
ethical only if the student has completed at least one graduate-level course in psychological testing.

D.
ethical only if Dr. Jones provides the student with appropriate training and supervision.

A

The minimum competence requirements for the administration of psychological tests are not well-defined in the Ethics Code or Standards for Educational and Psychological Testing. However, Standard 2.05 states that “Psychologists who delegate work to employees, supervisees, or research or teaching assistants or who use the services of others, such as interpreters, take reasonable steps to (1) avoid delegating such work to persons who have a multiple relationship with those being served that would likely lead to exploitation or loss of objectivity; (2) authorize only those responsibilities that such persons can be expected to perform competently on the basis of their education, training, or experience, either independently or with the level of supervision being provided; and (3) see that such persons perform these services competently.”

a. Incorrect APA’s ethical guidelines do not require that psychological tests be administered by licensed professionals only.
b. Incorrect Dr. Jones should, of course, be responsible for the activities of his assistant. However, this is not sufficient.
c. Incorrect The completion of a single course in psychological testing would probably be insufficient training for the administration of psychological tests.
d. CORRECT Of the alternatives given, this response is most consistent with the requirements of the Standard 2.05 of the Ethics Code and Principle II.7 of the Canadian Code of Ethics for Psychologists.

The correct answer is: ethical only if Dr. Jones provides the student with appropriate training and supervision.

152
Q

Research on complex clinical decision-making has confirmed that:
Select one:

A.
statistical prediction is usually more accurate than clinical judgment.

B.
clinical judgment is usually more accurate than statistical prediction.

C.
clinical judgment is more accurate than statistical prediction as long as the clinician “optimizes” rather than “satisfices.”

D.
clinical judgment and statistical prediction are about equal in accuracy.

A

Although the relative accuracy of statistical (actuarial) and clinical predictions continues to be debated, the research indicates that some general conclusions can be drawn.

a. CORRECT In their meta-analysis of the research, Grove and Meehl (1996) found that actuarial prediction alone was most accurate in 64 studies, a combination of actuarial prediction and clinical judgment was most accurate than either alone in 64 studies, and clinical judgment alone was most accurate in only 8 studies.
b. Incorrect See explanation for response a.
c. Incorrect See explanation for response a.
d. Incorrect See explanation for response a.

The correct answer is: statistical prediction is usually more accurate than clinical judgment.

153
Q

Jack and Joe are friends who both have recently taken jobs as life insurance salesmen and Jack is not certain that he is happy with his new job. When Jack is asked why he and Joe took these jobs, he is most likely to reply:
Select one:

A.
I did so because I needed the money; Joe did so because he likes sales.

B.
We both did so because we needed the money.

C.
We both did so because we like sales.

D.
I did so because I like sales; Joe did so because he needed the money.

A

People tend to attribute the behavior of others to dispositional (internal) factors, and this tendency is known as the “fundamental attribution bias.” Moreover, people are likely to attribute their own behavior to situational (external) factors; this tendency is known as the “actor-observer effect.” (One exception to the actor-observer effect is that individuals are likely to attribute their behavior to dispositional factors when the behavior has positive consequences, and this tendency is known as “self-serving bias.”)

a. CORRECT This response is most in line with the fundamental attribution bias and the actor-observer effect. Jack is attributing his own behavior to an external, situational factor and Joe’s behavior to an internal, dispositional factor.
b. Incorrect See explanation for response a.
c. Incorrect See explanation for response a.
d. Incorrect See explanation for response a.

The correct answer is: I did so because I needed the money; Joe did so because he likes sales.

154
Q

When conducting a research study, you want to ensure that you will detect a difference between the treatment group and the control (no treatment) group. Therefore, you will:
Select one:

A.
decrease error variance by decreasing the magnitude of the independent variable.

B.
increase experimental variance by increasing the magnitude of the independent variable.

C.
decrease the probability of making a Type I error by decreasing alpha.

D.
decrease the probability of making a Type II error by increasing beta.

A

Rephrased, this question would read “how do you increase power?” To identify the correct answer to this question, your understanding of power must be beyond a definitional level (e.g., “power is 1 - beta,” or “power is the probability of correctly rejecting a false null hypothesis,”) and at an abstract or conceptual level. Power can be thought of as the sensitivity of an empirical study. That is, if significant differences exist between groups, a powerful study will be sensitive enough to detect the differences.

a. Incorrect Decreasing the magnitude of the independent variable will result in less power. For instance, if you are studying the effects of a drug on a behavior, you are less likely to detect its effects when your experimental group receives a low dosage than when it receives a higher dosage. Moreover, this response does not make sense because decreasing the magnitude of the independent variable does not decrease error variance.
b. CORRECT Increasing the magnitude of the independent variable would increase experimental variance by increasing the difference between the experimental and control (no treatment) groups. This would increase power, or the ability to detect the effects of the independent variable. As an example, if you are studying the effects of a new learning procedure on performance, you are more likely to detect its effects if you administer the learning procedure to the experimental group subjects for six weeks rather than for one week.
c. Incorrect Decreasing alpha does decrease the probability of making a Type I error but, at the same time, it reduces power.
d. Incorrect Power does increase as the probability of making a Type II error decreases. However, this response does not make sense, since an increase in beta will, by definition, result in an INCREASE of the probability of making a Type II error.

The correct answer is: increase experimental variance by increasing the magnitude of the independent variable.

155
Q

To maximize the cueing function of feedback regarding goal achievement:
Select one:

A.
internal and external rewards must be available when goals are achieved.

B.
the feedback must be specific (rather than global) in nature.

C.
the feedback should be outcome (versus process) oriented.

D.
the level of behavior required to achieve the goal must be attainable.

A

This is an example of the “distant galaxy” questions that occasionally appear on the licensing exam. Note that the best strategy for these questions is to use what you know and to avoid being enticed by responses that “sound impressive.”

a. Incorrect Rewards are important in goal achievement but are associated more with the motivating function of feedback than its cueing function.
b. CORRECT Common sense probably suggests that this is the correct answer and, in fact, it is. Feedback is more effective when it is specific than when it is global or general.
c. Incorrect To maximize its effectiveness, feedback should provide information about both outcome and process.
d. Incorrect This is also important but, again, is more relevant to the motivating function of feedback. (See, e.g., D. A. Nadler, Feedback and organizational development: Using data-based methods, Reading, MA: Addison-Wesley, 1977.)

The correct answer is: the feedback must be specific (rather than global) in nature.

156
Q

When using the Premack Principle to increase a desirable behavior, the reinforcer is:
Select one:

A.
a stimulus that naturally elicits the target behavior.

B.
a stimulus that can be applied intermittently following the target behavior.

C.
a generalized conditioned reinforcer.

D.
a high probability behavior.

A

The Premack Principle is an application of positive reinforcement.

a. Incorrect See explanation for response d.
b. Incorrect See explanation for response d.
c. Incorrect See explanation for response d.
d. CORRECT When using the Premack Principle, a high probability behavior is used to reinforce a low probability behavior in order to increase the frequency of the low probability behavior.

The correct answer is: a high probability behavior.

157
Q

A study conducted by Buhrmester and Furman (1990) of 3rd, 6th, 9th, and 12th graders found that, as children approach the end of middle childhood, relationships between siblings become more:
Select one:

A.
distant.

B.
egalitarian.

C.
conflictual.

D.
complementary.

A

Research by D. Buhrmester and W. Furman found a combination of closeness and conflict among siblings in middle childhood with a trend toward less conflict and greater egalitarianism with increasing age (Perceptions of sibling relationships during middle childhood and adolescence, Child Development, 61, 1387-1398, 1990).

a. Incorrect See explanation for response b.
b. CORRECT The study cited in the question found that, by the end of middle childhood, sibling relationships had become much more egalitarian and lower in intensity and conflict.
c. Incorrect See explanation for response b.
d. Incorrect See explanation for response b.

The correct answer is: egalitarian.

158
Q

_________________ has been used as a pharmacologic model for Schizophrenia because drugs that alleviate the former also reduce the symptoms of the latter.
Select one:

A.
Serotonin syndrome

B.
Neuroleptic malignant syndrome

C.
Alcohol delirium tremens

D.
Amphetamine psychosis

A

Answer D is correct: Research on amphetamine psychosis has provided support for the dopamine hypothesis for Schizophrenia: First, amphetamines exert their effects by altering dopamine activity. Second, a high dose of amphetamine produces symptoms similar to those associated with Schizophrenia, and amphetamines exacerbate the symptoms of Schizophrenia. Third, drugs that reduce dopamine levels alleviate the symptoms of amphetamine psychosis and Schizophrenia.

Answers A, B, and C: See explanation for answer D.

The correct answer is: Amphetamine psychosis

159
Q

In group therapy, members establish a “taking turns” pattern in which each group session is devoted, sequentially, to each group member. In response to this situation, Yalom (1985) recommends:
Select one:

A.
mass group interpretation.

B.
here-and-now activation.

C.
paradoxical prescription.

D.
reframing.

A

In his book, The Theory and Practice of Group Psychotherapy, Yalom refers to the situation described in this question as an example of antitherapeutic group norms.

a. CORRECT Yalom considers such turn-taking as an obstacle to group therapy (e.g., because it may force some members to self-disclose prematurely) and recommends that it be dealt with through the use of mass group interpretation. This involves discussing the problem and its potential for negative effects on group members and the group process.
b. Incorrect Here-and-now activation is a technique advocated by Yalom, but it is not relevant to the situation described in the question. (Here-and-now activation is useful when a group member is focusing on past events in his/her life.)
c. Incorrect The use of paradoxical prescription is not something that is advocated by Yalom for this type of situation.
d. Incorrect Reframing is not a technique advocated by Yalom for this type of situation.

The correct answer is: mass group interpretation.

160
Q

The failure to take into account the prior probability of an event when considering its current probability is referred to as:
Select one:

A.
the base-rate fallacy.

B.
the fundamental attribution bias.

C.
illusory correlation.

D.
the discounting principle.

A

The correct answer to this question is the term that sounds like what it is – the “base-rate fallacy.”

a. CORRECT The base-rate fallacy is the failure to take the base rate into account when making decisions or forming attitudes. An example of the base-rate fallacy is the tendency of people to be more afraid of flying than of driving a car despite the fact that more people are killed on the road than in the sky.
b. Incorrect The fundamental attribution bias is described in the Social Psychology chapter of the written study materials.
c. Incorrect Illusory correlation is also described in the Social Psychology chapter.
d. Incorrect The discounting principle refers to the tendency to decrease the role of a given cause in producing an effect when other plausible causes are present.

The correct answer is: the base-rate fallacy.

161
Q

According to __________, an employee’s motivation is a function of the degree to which the employee perceives that his/her job inputs and outcomes are comparable to the inputs and outcomes of others performing similar jobs.
Select one:

A.
equity theory

B.
social-cognitive theory

C.
expectancy theory

D.
two-factor theory

A

For the exam, you want to be familiar with the four theories of motivation listed in the answers to this question. These are described in the Industrial-Organizational Psychology chapter of the written study materials.

a. CORRECT Equity theory emphasizes the impact of social comparison on motivation and focuses on an employee’s comparison of his/her input/outcome ratio to the input/outcome ratios of others doing the same or similar work.
b. Incorrect See explanation for response a.
c. Incorrect See explanation for response a.
d. Incorrect See explanation for response a.

The correct answer is: equity theory

162
Q

When using the __________ to assess level of consciousness, the patient’s score can range from 3 to 15 with scores of 3 to 8 indicating an unconscious state.
Select one:

A.
Glasgow Coma Scale

B.
Mini Mental State Exam

C.
Beery-Buktenica Test

D.
Rancho Scale

A

For the exam, you want to be familiar with the purposes of each of the tests listed in the answers to this question. This information is provided in the Psychological Assessment chapter of the written study materials.

a. CORRECT This answer accurately describes the possible range of scores on the Glasgow scale.
b. Incorrect Scores on the MMSE range from 0 to 30.
c. Incorrect The Beery-Buktenica is a measure of visual-motor integration.
d. Incorrect The Rancho Scale is used to evaluate recovery following traumatic brain injury and involves rating the individual in terms of 10 levels of functioning.

The correct answer is: Glasgow Coma Scale

163
Q

In Ainsworth’s Strange Situation, children classified as __________may or may not cry when their mother leaves, ignore her when she returns, and, if they do become distressed, are just as likely to be comforted by a stranger as by their own mother.
Select one:

A.
secure

B.
insecure/avoidant

C.
insecure/resistant

D.
disorganized/disoriented

A

Ainsworth originally identified three attachment patterns –secure, insecure/avoidant, and insecure/resistant. Main subsequently added the disorganized/disoriented pattern.

a. Incorrect Secure infants often exhibit some distress when mom leaves and are unlikely to be comforted by a stranger.
b. CORRECT Insecure/avoidant infants ordinarily show little distress in response to mom’s departure, ignore her when she returns, and are often more friendly to a stranger than to mom.
c. Incorrect Insecure/resistant infants become very upset when mom leaves and are not comforted by her return.
d. Incorrect Disorganized/disoriented infants exhibit contradictory, disoriented behavior during the events of the Strange Situation.

The correct answer is: insecure/avoidant

164
Q

When using covert sensitization to reduce a client’s cigarette smoking, a mild electric shock is applied to the client’s hand just as he begins to smoke. In this situation, the electric shock is acting as a(n):
Select one:

A.
conditioned stimulus.

B.
unconditioned stimulus.

C.
negative reinforcer.

D.
negative discriminative stimulus.

A

Knowing that covert sensitization is classified as a type of aversive counterconditioning may have helped you identify the correct answer to this question.

a. Incorrect See explanation for response b.
b. CORRECT When using covert sensitization, a stimulus that is associated with the undesirable behavior is the conditioned stimulus, which is paired with an unconditioned stimulus that naturally elicits an unpleasant response. In the situation described in this question, the cigarette is the conditioned stimulus and the electric shock is the unconditioned stimulus.
c. Incorrect See explanation for response b.
d. Incorrect See explanation for response b.

The correct answer is: unconditioned stimulus.

165
Q

A message is likely to be most persuasive when:
Select one:

A.
the message is in the listener’s latitude of noncommitment and there is a large discrepancy between the listener’s initial position and the position advocated by the message.

B.
the message is in the listener’s latitude of noncommitment and there is a small to moderate discrepancy between the listener’s initial position and the position advocated by the message.

C.
the message is in the listener’s latitude of acceptance and there is a moderate discrepancy between the listener’s initial position and the position advocated by the message.

D.
the message is in the listener’s latitude of acceptance and there is a large discrepancy between the listener’s initial position and the position advocated by the message.

A

This question requires you to be familiar with Social Judgment Theory, which is described in the Social Psychology chapter of the written study materials. Answer C is correct: The research has found that people are most likely to be persuaded when the message targets their latitude of acceptance (i.e., the message represents an opinion that a listener finds tolerable and would be willing to consider) and when there is a small to moderate discrepancy between their initial position and the position advocated by the message.

The correct answer is: the message is in the listener’s latitude of acceptance and there is a moderate discrepancy between the listener’s initial position and the position advocated by the message.

166
Q

To investigate diagnostic overshadowing, a researcher will design a study that examines:
Select one:

A.
whether a therapist’s knowledge that a client has a Mood Disorder reduces the likelihood that the therapist will consider or recognize that the client also has another disorder.

B.
how a therapist’s prior experience with clients with a Mood Disorder affects the likelihood that she will consider a Mood Disorder first when assigning a diagnosis to a new client.

C.
the likelihood that a person who has a family member with a Mood Disorder will interpret his or her own symptoms as indicating a Mood Disorder.

D.
the impact of a therapist’s knowledge of the severity of a client’s Mood Disorder on the therapist’s predictions about the client’s therapy outcomes.

A

The term diagnostic overshadowing was initially used to describe the tendency of health professionals to attribute all behavioral, emotional, and social problems of individuals with an Intellectual Disability to that diagnosis.

a. CORRECT The study described in this answer would address the effects of having one diagnosis on the likelihood that other diagnoses would be considered and, therefore, would be useful for investigating diagnostic overshadowing.
b. Incorrect See explanation above.
c. Incorrect See explanation above.
d. Incorrect See explanation above.

The correct answer is: whether a therapist’s knowledge that a client has a Mood Disorder reduces the likelihood that the therapist will consider or recognize that the client also has another disorder.

167
Q

Conger’s (1956) tension reduction hypothesis emphasizes the role of which of the following in substance addiction?
Select one:

A.
anxiety and other negative emotions

B.
environmental cues

C.
higher-order conditioning

D.
heredity

A

Answer A is correct: According to J. J. Conger, people drink to alleviate anxiety, fear, and other negative emotions and are reinforced for doing so because alcohol does, in fact, reduce tension (Reinforcement theory and the dynamics of alcoholism, Quarterly Journal of Studies on Alcohol, 17, 296-306, 1956).

Answers B, C, and D: See explanation for answer A.

The correct answer is: anxiety and other negative emotions

168
Q

Sue and Sue (2003) propose that culturally competent therapists work toward achieving goals in which of the following domains?
Select one:

A.
awareness, knowledge, and skills

B.
integrity, flexibility, and openness to experience

C.
thoughts, feelings, and actions

D.
self-understanding, empathy, and expertise

A

In their book, Counseling the Culturally Diverse: Theory and Practice, D.W. Sue and D. Sue (2003), describe the characteristics of culturally competent practitioners.

a. CORRECT According to Sue and Sue, culturally competent practitioners work toward achieving self-awareness, cultural knowledge, and appropriate skills.
b. Incorrect See explanation above.
c. Incorrect See explanation above.
d. Incorrect See explanation above.

The correct answer is: awareness, knowledge, and skills

169
Q

To evaluate the effectiveness of a training program designed to promote safe behaviors at work, Dr. Juarez observes and records each employee’s behavior for 30 minutes each day for one week before and one week after his/her participation in the training program. Dr. Juarez is using which of the following research designs?
Select one:

A.
multiple baseline

B.
changing criterion

C.
time sampling

D.
time series

A

Dr. Juarez’s study involves measuring the safety behaviors of workers at regular intervals before and after their participation in the training program.

a. Incorrect A multiple baseline design is single-case (single-subject) design that involves sequentially applying a treatment across different baselines - i.e., across participants, behaviors, or settings.
b. Incorrect The changing criterion design is also a single-case design. When using this design, the criterion (e.g., the desired frequency of the target behavior) is progressively increased or decreased during the course of the study.
c. Incorrect Time sampling is a behavioral observation technique that involves observing an individual or group of individuals during prespecified periods of time and recording whether or not the target behavior(s) occurred during each period.
d. CORRECT When using a time series design, the dependent (outcome) variable is assessed several times at regular intervals before and after administration of the independent variable.

The correct answer is: time series

170
Q

Five participants in a research study work individually at computer terminals to generate solutions to novel problems. Each participant in the study simultaneously reviews the suggested solutions of other participants while entering his or her own solutions. The purpose of this study is to assess the effects of technology on:
Select one:

A.
social inhibition.

B.
groupthink.

C.
quality circles.

D.
brainstorming.

A

The phrase “generate solutions to novel problems” may have helped you identify the correct answer to this question.

a. Incorrect Social inhibition occurs when an individual’s performance on a task is negatively affected by the mere presence of other individuals.
b. Incorrect Groupthink is a suspension of critical thinking that may occur in highly cohesive groups.
c. Incorrect Quality circles are small voluntary groups of employees who work together and meet regularly to discuss and resolve work-related problems.
d. CORRECT Brainstorming was originally developed as a way to improve group performance on difficult or novel tasks. Although the research on brainstorming has generally found that people come up with more and better solutions when working alone than when working as a group, there is some evidence that brainstorming by computer improves its outcomes.

The correct answer is: brainstorming.

171
Q

Based on the research, the best conclusion that can be drawn about the impact of biological sex and gender-role identity on self-esteem is that:
Select one:

A.
biological sex has a greater impact than gender role, with male (versus female) sex being associated with higher levels of self-esteem.

B.
biological sex has a greater impact than gender role, with female (versus male) sex being associated with higher levels of self-esteem.

C.
gender role has a greater impact than biological sex, with androgyny being associated with the highest levels of self-esteem.

D.
gender role has a greater impact than biological sex, with masculinity in males and femininity in females being associated with the highest levels of self-esteem.

A

The original research on the impact of gender-role identity on self-esteem was conducted by J. A. Hall and A. G. Halberstadt [Masculinity and femininity in children: Development of the Children’s Personal Attributes Questionnaire, Developmental Psychology, 16, 270-280, 1980].

a. Incorrect Gender-role identity has been found to have a greater impact than biological sex on self-esteem in children.
b. Incorrect See explanation for responses a and c.
c. CORRECT The study by Hall and Halberstadt and subsequent studies have generally confirmed that androgyny (which combines masculine and feminine characteristics and preferences) is associated with the highest levels of self-esteem in both boys and girls. Some studies have also found that masculinity, to a somewhat lesser degree, is associated with higher levels of self-esteem than femininity in both boys and girls.
d. Incorrect See explanation for response c.

The correct answer is: gender role has a greater impact than biological sex, with androgyny being associated with the highest levels of self-esteem.

172
Q

Eighteen-month old Tylee has learned the word “cup” and applies it to other similar objects such as bowls, glasses, and bottles. This is referred to as:
Select one:

A.
underextension.

B.
overextension.

C.
overregularization.

D.
expansion.

A

Children exhibit a number of predictable errors when learning to speak.

a. Incorrect Underextension involves applying a term too narrowly (e.g., using “dog” to refer only to the family dog).
b. CORRECT As its name implies, overextension involves applying a word to a wider collection of objects or events than is appropriate.
c. Incorrect Overregularization is an overextension of grammatical rules to words that are exceptions (e.g., adding “s” to “feet”).
d. Incorrect Expansion refers to the feedback that adults seem to naturally give young children, which provides children with information on appropriate language use.

The correct answer is: overextension.

173
Q

According to ____________, symbolic play offers young children opportunities to engage in activities that are developmentally more advanced than they could successfully perform in reality on their own.
Select one:

A.
J. H. Flavell

B.
E. Gibson

C.
L. Vygotsky

D.
J. Piaget

A

Symbolic (make-believe) play refers to play in which one object represents another (e.g., a doll represents a baby).

a. Incorrect Flavell is probably best known for his research on children’s ability to distinguish between appearance and reality.
b. Incorrect Eleanor and James Gibson are best known for their research on perceptual development in infants.
c. CORRECT For Vygotsky, symbolic play provides a “zone of proximal development” that enables a child to practice behaviors in a situation that requires less precision and accuracy than would be required in reality.
d. Incorrect Piaget viewed symbolic play as an opportunity to practice representational schemes.

The correct answer is: L. Vygotsky

174
Q

You saw a family in therapy for several months but they terminated when the husband and wife decided to get a divorce. Now, two months later, you are being asked to conduct an evaluation of the husband for a custody hearing. Your best course of action is to:
Select one:

A.
do so but avoid making a recommendation without evaluating all members of the family.

B.
do so only with the consent of both parents.

C.
do so only if you believe you can remain unbiased.

D.
not conduct the evaluation.

A

This issue is addressed by ethical guidelines that require psychologists to avoid multiple relationships.

a. Incorrect See explanation for response d.
b. Incorrect See explanation for response d.
c. Incorrect See explanation for response d.
d. CORRECT This issue is addressed, for example, in Guideline 7 of the Guidelines for Child Custody Evaluations in Family Law Proceedings, which states that “psychologists strive to avoid conflicts of interest and multiple relationships in conducting evaluations.”

The correct answer is: not conduct the evaluation.

175
Q

________ was among the first researchers to propose the existence of a neural circuit that mediates the experience and expression of emotion.
Select one:

A.
Sperry

B.
Fechner

C.
Papez

D.
Penfield

A

The circuit referred to in this question is known as Papez’s circuit.

a. Incorrect Sperry conducted research on split-brain patients.
b. Incorrect Fechner developed Fechner’s Law, which is one of several laws used to explain the relationship between sensation and perception.
c. CORRECT Papez concluded that destruction of certain interconnected regions in the brain has an adverse impact on emotional experience and expression. These regions include the mammillary bodies, anterior thalamus, cingulate cortex, hippocampus, and fornix.
d. Incorrect Penfield found that stimulation of certain brain areas produced certain movements, while stimulation of other areas produced certain sensations.

The correct answer is: Papez

176
Q

An infant’s first expression of fear is often in response to unfamiliar adults. The onset and severity of this “stranger anxiety” depends on several factors – for instance, the child’s temperament and previous experience with strangers – but is ordinarily first evident when the child is _____ months of age.
Select one:

A.
4 to 6

B.
8 to 10

C.
10 to 12

D.
13 to 15

A

Stranger anxiety is considered one manifestation of a child’s attachment to his/her primary caregiver(s).

a. Incorrect See explanation for response b.
b. CORRECT In most infants, stranger anxiety is first evident by about 8 to 10 months.
c. Incorrect See explanation for response b.
d. Incorrect See explanation for response b.

The correct answer is: 8 to 10

177
Q

A college freshman is being pressured by his parents to make a career choice and choose a college major. The young man is leaning toward a career in agriculture, but both of his parents are accountants and they want him to choose accounting so that he can eventually take over the family business. In terms of Marcia’s “identity statuses,” if the young man acquiesces to his parents’ wishes, this will represent:
Select one:

A.
moratorium.

B.
diffusion.

C.
foreclosure.

D.
pseudo-achievement.

A

Marcia (1980) distinguished between four identity statuses: diffusion, foreclosure, moratorium, and achievement. Information about these statuses is provided in the Lifespan Development chapter of the written study materials.

a. Incorrect See explanation for response c.
b. Incorrect See explanation for response c.
c. CORRECT Identity-foreclosed individuals have made a commitment to an identity that has been imposed by parents or other authority figures.
d. Incorrect See explanation for response c.

The correct answer is: foreclosure.

178
Q

The progression of Alzheimer’s disease is often described in terms of three stages. Which of the following symptoms are characteristic of the first stage?
Select one:

A.
anterograde amnesia, tremor or clumsiness, apathy, and alexia

B.
anterograde amnesia, wandering, irritability or sadness, and anomia

C.
anterograde and retrograde amnesia, ideomotor apraxia, restlessness, and labile mood

D.
retrograde amnesia, labile mood, ideomotor apraxia, and wandering

A

Answer B is correct: The symptoms listed in this answer are characteristic of the initial stage of Alzheimer’s disease. Some or all of the symptoms listed in the other answers are either characteristic of the second or third stage or are not common symptoms of this disorder. (A description of the stages is provided in the Abnormal Psychology chapter of the written study materials.)

Answers A, C, and D: See explanation for answer B.

The correct answer is: anterograde amnesia, wandering, irritability or sadness, and anomia

179
Q
Hans Eysenck (1952) concluded that the apparent effectiveness of psychotherapy is due to:
Select one:

A.
patient expectancies.

B.
nonspecific factors of therapy.

C.
spontaneous remission.

D.
a false consensus effect.

A

Eysenck’s conclusions sparked an ongoing controversy in the literature about the effectiveness of psychotherapy.

a. Incorrect See explanation for response c.
b. Incorrect See explanation for response c.
c. CORRECT Eysenck’s review of the research led him to conclude that the effects of psychotherapy are small or nonexistent and are due primarily to spontaneous remission.
d. Incorrect See explanation for response c.

The correct answer is: spontaneous remission.

180
Q

A parent yells at her 5-year-old son whenever he misbehaves. The boy quickly figures out that, if he apologizes for his misbehavior, his mother will stop yelling at him. Consequently, he begins to say, “I’m sorry” as soon as his mother starts yelling at him. The boy’s apologizing is best described as the result of which of the following?
Select one:

A.
avoidance conditioning

B.
escape conditioning

C.
stimulus control

D.
stimulus generalization

A

In this situation, the boy’s apologizing is maintained because it allows him to “escape” his mother’s yelling once it has started.

a. Incorrect Avoidance conditioning involves presenting a stimulus that signals that an aversive event is about to occur so that eventually the stimulus elicits a behavior that allows the individual to avoid the aversive event.
b. CORRECT In this situation, the boy doesn’t entirely avoid his mother’s yelling, but he does “escape” it once it has started by apologizing.
c. Incorrect A behavior is brought under stimulus control when a stimulus signals whether or not the behavior will be reinforced.
d. Incorrect Stimulus generalization occurs when stimuli similar to the original CS elicit the CR.

The correct answer is: escape conditioning

181
Q

Cross-cultural research has identified several universal emotions that are associated with similar facial expressions by individuals from diverse cultures. Which of the following is NOT one of these?
Select one:

A.
fear

B.
anger

C.
disgust

D.
shame

A

The universal emotions identified by Ekman (1993) and Izard (1971) are: happiness, sadness, fear, anger, disgust, surprise, and contempt. People from diverse cultural backgrounds use very similar facial expressions to express these emotions.

a. Incorrect See list above.
b. Incorrect See list above.
c. Incorrect See list above.
d. CORRECT Shame is not one of the universal emotions identified by the researchers.

The correct answer is: shame

182
Q

Super’s life-career rainbow depicts the relationship between the individual’s life stages and his/her:
Select one:

A.
level of career differentiation.

B.
stage of ego identity development.

C.
career interests.

D.
life roles.

A

Super’s life-space, life-span theory of career development integrates several concepts, and his life-career rainbow depicts the relationship between two of these concepts.

a. Incorrect Differentiation is a term used by Holland to describe the clarity of a person’s vocational personality type. According to Holland, a person has a high degree of differentiation when he/she obtains a high score on one personality type and low scores on the other types.
b. Incorrect Ego identity is a focus of Tiedeman and O’Hara’s career identity development model.
c. Incorrect Career interests are not depicted in Super’s life-career rainbow.
d. CORRECT Super used the life-career rainbow to depict the impact of a person’s life stage and life roles on his/her career development.

The correct answer is: life roles.

183
Q

Bouchard and McGue (1981) reported a correlation coefficient of .45 for the IQ test scores of:
Select one:

A.
biological parent and child (together).

B.
biological parent and child (apart).

C.
biological siblings raised together.

D.
biological siblings raised apart.

A

For the exam, you want to be familiar with the correlation coefficients reported by Bouchard and McGue (1981). These are provided in the Psychological Assessment chapter of the written study materials.

a. Incorrect The reported correlation for a biological parent and his/her child when the child is raised by that parent is .39.
b. Incorrect The reported correlation for a biological parent and his/her child when the child is not raised by that parent is .22.
c. CORRECT Bouchard and McGue reported a correlation of .45 for biological siblings reared together.
d. Incorrect These investigators reported a correlation of .24 for biological siblings reared apart.

The correct answer is: biological siblings raised together.

184
Q

Which of the following are required for a DSM diagnosis of Antisocial Personality Disorder?
Select one:

A.
a pervasive pattern of disregard for and violation of the rights of others that began prior to age 15 plus a current age of 16 or older

B.
a pervasive pattern of disregard for and violation of the rights of others that began prior to age 15 plus a current age of 18 or older

C.
a pervasive pattern of disregard for and violation of the rights of others since age 14, a history of Conduct Disorder symptoms prior to age 14, plus a current age of 16 or older

D.
a pervasive pattern of disregard for and violation of the rights of others since age 15, a history of Conduct Disorder symptoms prior to age 15, plus a current age of 18 or older

A

Answer D is correct: For the diagnosis of Antisocial Personality Disorder, the DSM requires a pervasive pattern of disregard for and violation of the rights of others since age 15 with symptoms of Conduct Disorder prior to age 15 and a current age of 18 years or older.

Answers A, B, and C: See explanation for answer D.

The correct answer is: a pervasive pattern of disregard for and violation of the rights of others since age 15, a history of Conduct Disorder symptoms prior to age 15, plus a current age of 18 or older

185
Q

The use of implosive therapy is based on the assumption that:
Select one:

A.
pairing stimuli associated with an undesirable response with a stimulus that produces a more adaptive response will result in elimination of the undesirable response.

B.
consistently withholding reinforcement from an undesirable conditioned response will eventually result in extinction of that response.

C.
repeatedly presenting stimuli associated with an undesirable conditioned response without the unconditioned stimulus will eventually result in extinction of that response.

D.
consistently applying an aversive (unconditioned) stimulus following an undesirable conditioned response will eventually eliminate that response

A

Knowing that implosive therapy is based on classical conditioning and uses extinction to eliminate an undesirable response would have helped you identify the correct answer to this question.

a. Incorrect See explanation for response c.
b. Incorrect See explanation for response c.
c. CORRECT Implosive therapy is used to treat phobic responses and involves repeatedly presenting the feared stimulus (conditioned stimulus) in imagination without the unconditioned stimulus so that the feared stimulus no longer produces an anxiety response. It also incorporates psychodynamic principles by embellishing the imagined images with psychodynamic themes. Additional information about implosive therapy is provided in the Learning Theory chapter of the written study materials.
d. Incorrect See explanation for response c.

The correct answer is: repeatedly presenting stimuli associated with an undesirable conditioned response without the unconditioned stimulus will eventually result in extinction of that response.

186
Q

In the context of test validity, specificity refers to which of the following?
Select one:

A.
the correct identification of true negatives

B.
the correct identification of true positives

C.
the correct identification of true negatives and true positives

D.
the correct identification of false negatives and false positives

A

As described in the Test Construction chapter of the written study materials, the data used to evaluate a test’s accuracy with regard to decision-making can be expressed in terms of sensitivity, specificity, positive predictive value, and negative predictive value.

a. CORRECT Specificity refers to the correct identification of true negatives – i.e., the percent of cases in the validation sample who do not have the characteristic being screened for and were accurately identified by the predictor as not having that characteristic.
b. Incorrect Sensitivity refers to the correct identification of true positives.
c. Incorrect See explanation above.
d. Incorrect See explanation above.

The correct answer is: the correct identification of true negatives

187
Q

The results of the Stroop Color-Word Association Test provide information on the examinee’s:
Select one:

A.
visual-motor integration.

B.
working memory.

C.
field dependence/independence.

D.
response inhibition.

A

The Stroop Color-Word Association Test was developed as a measure of response inhibition.

a. Incorrect See explanation for response d.
b. Incorrect See explanation for response d.
c. Incorrect See explanation for response d.
d. CORRECT When administering the Stroop Test, the examinee is presented with a list of color names that are printed in different colors (e.g., the word “red” might be printed in blue ink), and the examinee must say the color of the ink (“blue”). Because the printed name is the prepotent response, this test provides information on response inhibition.

The correct answer is: response inhibition.

188
Q

Holland’s Self-Directed Search provides scores on six occupational themes. A person who obtains the highest score on the realistic theme is best suited for a job involving:
Select one:

A.
physical, mechanical, or outdoor activities.

B.
scientific, mathematical, or analytic tasks.

C.
activities that require attention to detail and good organization skills.

D.
tasks that require business, management, or sales skills.

A

Holland’s scale distinguishes between five occupational themes (“RIASEC”): realistic, investigative, artistic, social, enterprising, and conventional. Additional information on these themes is provided in the Psychological Assessment chapter of the written study materials.

a. CORRECT People who score high on the realistic scale are well suited for these types of tasks.
b. Incorrect These tasks fit the investigative type.
c. Incorrect These activities fit the interests of individuals receiving the highest score on the conventional scale.
d. Incorrect These tasks are of greatest interest to individuals with a high score on the enterprising scale.

The correct answer is: physical, mechanical, or outdoor activities.

189
Q

You have been hired by a school psychologist to help enhance her ability to work with children with conduct problems. During the consultation, you realize that the psychologist is exhibiting “theme interference” - that is, she has a marked lack of objectivity when working with conduct-disordered children due to her own childhood experience with an emotionally-disturbed sibling. The type of consultation you are involved in is best described as:
Select one:

A.
process-centered.

B.
advocacy-centered.

C.
client-centered case consultation.

D.
consultee-centered case consultation.

A

In this situation, you have been hired by the school psychologist to help her deal more effectively with a particular type of client.

a. Incorrect Process consultation is a type of organizational consultation and development that focuses on communication and other employee processes.
b. Incorrect The purpose of advocacy consultation is to act as an advocate for a disenfranchised group.
c. Incorrect In client-centered case consultation, the focus of the consultation is on a specific client (not a group of clients).
d. CORRECT In consultee-centered case consultation, the focus is on the consultee’s skill, knowledge, or objectivity, and the goal is to improve the consultee’s effectiveness with members of a particular population.

The correct answer is: consultee-centered case consultation.

190
Q

According to Selye (1956), during the ______ stage of the general adaptation syndrome, the hypothalamus signals the pituitary gland to release adrenocorticotropic hormone (ACTH), which causes the adrenal cortex to release cortisol.
Select one:

A.
alarm

B.
resistance

C.
preparation

D.
exhaustion

A

Selye’s general adaptation sydrome consists of three stages – alarm, resistance, and exhaustion. Additional information about these stages is provided in the Physiological Psychology and Psychopharmacology chapter of the written study materials.

a. Incorrect See explanation for response b.
b. CORRECT The resistance stage occurs when stress persists and, as a result, the hypothalamus signals the pituitary gland to release ACTH which then activates the adrenal cortex to release the stress hormone cortisol.
c. Incorrect See explanation for response b.
d. Incorrect See explanation for response b.

The correct answer is: resistance

191
Q

In an experiment on memory, the researcher utters a string of ten letters within a span of 3 seconds and suddenly stops and asks the participant what the last four letters were. Apparently, this researcher is investigating:
Select one:

A.
working memory.

B.
semantic memory.

C.
iconic memory.

D.
echoic memory.

A

In this study, the participant must recall what he/she has just heard.

a. Incorrect Working memory is part of short-term memory.
b. Incorrect Semantic memory is part of long-term memory.
c. Incorrect Iconic memory is the visual aspect of sensory memory.
d. CORRECT The participant is being asked to recall what he/she has just heard – i.e., to recall information that in sensory memory. The auditory part of sensory memory is referred to as echoic memory.

The correct answer is: echoic memory.

192
Q

Korsakoff syndrome is caused by:
Select one:

A.
a lack of blood flow to the brain.

B.
inadequate intake or absorption of thiamine.

C.
damage to the somatosensory cortex.

D.
a build-up of cerebrospinal fluid in the ventricles.

A

Korsakoff syndrome is characterized by severe anterograde amnesia, retrograde amnesia, and confabulation.

a. Incorrect See explanation for response b.
b. CORRECT Korsakoff syndrome is caused by a thiamine deficiency, often as the result of chronic alcoholism, and involves atrophy of neurons in certain areas of the thalamus and mammillary bodies.
c. Incorrect See explanation for response b.
d. Incorrect See explanation for response b.

The correct answer is: inadequate intake or absorption of thiamine.

193
Q

A response that was acquired through avoidance learning (conditioning) is unlikely to be extinguished because the avoidance response is:
Select one:

A.
negatively reinforced.

B.
positively reinforced.

C.
negatively punished.

D.
positively punished.

A

In avoidance conditioning, a neutral stimulus (CS) has been paired with a fear-arousing stimulus (US) so that the CS produces a fear response (CR); and, eventually, the individual learns that he/she can avoid fear by avoiding the CS. In this situation, the avoidance response is being negatively reinforced – avoiding the CS is reinforced because the avoidance behavior eliminates the fear response. As an example, assume that you live in San Francisco and developed a fear of the Golden Gate Bridge because there was an earthquake the last time you drove on it. Consequently, you no longer drive on the Golden Gate Bridge because avoiding the bridge allows you to avoid experiencing fear. In other words, your avoidance of the bridge is being negatively reinforced. (In this situation, the Golden Gate Bridge is the CS, and the earthquake is the US.)

a. CORRECT Because the individual avoids the CS, extinction trials do not occur. In other words, the individual never has an opportunity to experience presentation of the CS without the US, which means that the CR will not be extinguished. In the example, your conditioned fear of the Golden Gate Bridge will never be extinguished because you never drive on the bridge and, therefore, never experience being on the bridge without an earthquake.
b. Incorrect See explanation for response a.
c. Incorrect See explanation for response a.
d. Incorrect See explanation for response a.

The correct answer is: negatively reinforced.

194
Q

Research investigating the relationship between interest test scores and future occupational choice suggests that these tests have the highest predictive validity for:
Select one:

A.
lower-class people.
B.
middle-class people.
C.
upper-class people.
D.
lower-and upper-class people.
A

Socioeconomic status is one factor that has been found to affect the predictive validity of interest inventories.

a. Incorrect Lower-class people do not always have the opportunity to pursue occupations that coincide with their interests and frequently choose jobs that provide the greatest pay and security.
b. CORRECT Members of the middle-class usually have the greatest latitude when it comes to choosing an occupation and, therefore, are most likely to choose jobs that coincide with their interests.
c. Incorrect Upper-class individuals often choose occupations on the basis of family tradition or societal expectations rather than interests.
d. Incorrect See explanations above.

The correct answer is: middle-class people.

195
Q

Habit reversal training is most likely to be used to treat which of the following disorders?
Select one:

A.
Reactive Attachment Disorder

B.
Intermittent Explosive Disorder

C.
Transvestic Disorder

D.
Childhood Onset Fluency Disorder

A

Answer D is correct: Habit reversal training was originally designed as a treatment for nervous habits and motor tics and has also been found useful for treating Childhood Onset Fluency Disorder (stuttering). It incorporates awareness, relaxation, motivation, competing response, and generalization training.

Answers A, B, and C: See explanation for answer D.

The correct answer is: Childhood Onset Fluency Disorder

196
Q

A primary distinction between Freud and those psychologists who are collectively identified as “neo-Freudians” (e.g., Fromm, Horney, and Sullivan) is that the latter:
Select one:

A.
emphasized the “innate wisdom” of the individual.

B.
placed greater emphasis on the ego functions and the impact of social influences.

C.
extended the number of and placed greater emphasis on instinctual (unconscious) forces.

D.
adopted a teleological approach that viewed behavior as being “pulled” rather than “pushed.”

A

The neo-Freudians adopted many of Freud’s basic assumptions about personality development but placed less emphasis on id functions and greater emphasis on the functions of the ego.

a. Incorrect This best describes Carl Rogers’ view of personality.
b. CORRECT Fromm, for example, viewed the individual as the product of social forces, and Sullivan emphasized the interpersonal aspects of behavior.
c. Incorrect The “instinctual forces” are governed by the id. As noted above, the neo-Freudians placed greater emphasis on the ego functions.
d. Incorrect This describes Alfred Adler’s personality theory. Although Adler is sometimes categorized as a neo-Freudian, his approach differs substantially from that of other neo-Freudians and, therefore, this is not the best answer.

The correct answer is: placed greater emphasis on the ego functions and the impact of social influences.

197
Q

In therapy, an object relations family therapist would be most interested in which of the following?
Select one:

A.
distinguishing between positive and negative connotations

B.
integrating attitudes, feelings, and behaviors

C.
interpreting focused and contextual transferences

D.
expanding awareness and self-responsibility

A

Object relations family therapy is a psychodynamic approach and relies on many of the techniques associated with psychodynamic psychotherapy

a. Incorrect See explanation for response c.
b. Incorrect See explanation for response c.
c. CORRECT D. E. Scharff and J. S. Scharff note that addressing transferences and countertransferences is a primary focus of object relations family therapy (Object relations family therapy, Northvale, NJ, Jason Aronson, 1991). These authors also distinguish between two types of transference - focused and contextual - and state that the latter is particularly important in family therapy.
d. Incorrect See explanation for response c.

The correct answer is: interpreting focused and contextual transferences

198
Q

Alfred Adler believed that a client’s earliest memories provide information about his/her:
Select one:

A.
defense mechanisms.

B.
internal working models.

C.
basic mistakes.

D.
ego-state.

A

Adlerians use a variety of techniques to obtain information about a client’s style of life.

a. Incorrect See explanation for response c.
b. Incorrect See explanation for response c.
c. CORRECT Basic mistakes are distorted beliefs and attitudes that contribute to a person’s style of life and may be revealed in the client’s earliest memories.
d. Incorrect See explanation for response c.

The correct answer is: basic mistakes.

199
Q

Because of a series of misbehaviors, your adolescent daughter has been restricted from participating in a number of activities over the next few months. To increase her positive behaviors, you decide to remove a restriction each time she engages in one of the desired behaviors. This is an example of:
Select one:

A.
positive reinforcement.

B.
negative reinforcement.

C.
response cost.

D.
shaping.

A

In this situation, you are removing a stimulus in order to increase a behavior.

a. Incorrect Punishment is used to decrease a behavior.
b. CORRECT Removal of a stimulus (e.g., a restriction) after a behavior in order to increase that behavior is referred to as negative reinforcement.
c. Incorrect Response cost is a type of punishment and is used to decrease the occurrence of a behavior.
d. Incorrect Shaping is a type of positive reinforcement and involves APPLYING (not removing) a stimulus following a behavior in order to increase that behavior.

The correct answer is: negative reinforcement.

200
Q

In the context of the serial position effect, the “primacy effect”:
Select one:

A.
is due to transfer of information from short- to long-term memory.

B.
is due to interference that prohibits transfer from short- to long-term memory.

C.
is due to the repetition of information so that it is maintained in short-term memory.

D.
is due to distractions that make it difficult to retrieve information from long-term memory.

A

The serial position effect refers to the tendency to recall items in the beginning and end of a word list better than the items in the middle of the list.

a. CORRECT The ability to recall items in the beginning of the list (primacy effect) is believed to be due to the fact that these items have been transferred from short- to long-term memory, while the ability to recall items in the end of the list (recency effect) is due to the fact that they are still present in short-term memory.
b. Incorrect See explanation above.
c. Incorrect See explanation above.
d. Incorrect See explanation above.

The correct answer is: is due to transfer of information from short- to long-term memory.

201
Q

Dr. Bekke, a cognitive-behavioral therapist, is most likely to rely on which of the following when working with a client exhibiting symptoms of PTSD after exposure to a life-threatening traumatic event?
Select one:

A.
exposure and cognitive restructuring

B.
applied relaxation and overcorrection

C.
self-monitoring and self-control desensitization

D.
covert sensitization and stress inoculation

A

Answer A is correct: The treatment-of-choice for PTSD is a comprehensive cognitive-behavioral intervention that incorporates exposure, cognitive restructuring, and anxiety management or similar techniques.

Answers B, C, and D: See explanation for answer A.

The correct answer is: exposure and cognitive restructuring

202
Q

According to Murray Bowen, when two family members experience stress, they may recruit a third family member to reduce the tension in their relationship. Then, if the stress continues, the resulting tension may spread to other family members and, as more people become involved, _________ develops.
Select one:

A.
a destablized ("runaway") system

B.
a series of interlocking triangles

C.
emotional cutoff

D.
emotional flooding

A

Knowing that Bowen is associated with triangles would have helped you identify the correct answer to this question.

a. Incorrect In a system, positive feedback amplifies deviation or change and, in some situations, can produce a runaway system.
b. CORRECT Bowen proposed that, when a three-person system (triangle) does not reduce the stress between two family members, other individuals may be recruited, and the family system may eventually include a series of interlocking triangles.
c. Incorrect Emotional cutoff refers to extreme emotional distancing by a family member in an attempt to cut off ties from other members.
d. Incorrect Emotional flooding refers to uncontrollable and uncontrolled emotional expression.

The correct answer is: a series of interlocking triangles

203
Q

For a test with a mean of 100 and standard deviation of 15, the upper and lower limits of the standard error of measurement are:
Select one:

A.
0 to +1.0.

B.
-1.0 to +1.0.

C.
0 to 100.

D.
0 to 15.

A

Knowing the formula for the standard error of measurement would have helped you identify the correct answer to this question: The standard error of measurement equals the standard deviation times the square root of one minus the reliability coefficient.

a. Incorrect See explanation for response d.
b. Incorrect See explanation for response d.
c. Incorrect See explanation for response d.
d. CORRECT When a test’s reliability coefficient is equal to +1.0 (the highest reliability coefficient possible), there is no error in measurement; and the standard error of measurement equals zero. At the other extreme, when the test’s reliability coefficient is equal to 0, the standard error of measurement equals the standard deviation of the test scores.

The correct answer is: 0 to 15.

204
Q

Data from the National Health Care Survey (Raofi & Schappert, 2006) indicate that, in general, central nervous system drugs are most commonly provided, prescribed, or continued at ambulatory office visits by primary care physicians. An exception to this general rule are _________, which are most commonly provided, prescribed, or continued by practitioners in medical specialty offices.
Select one:

A.
antianxiety drugs

B.
antidepressants

C.
antipsychotic and antimanic drugs

D.
sedatives and hypnotics

A

This is a difficult question and one that you would not want too spend to much time trying to figure out if you are unfamiliar with the results of the National Health Care Survey.

a. Incorrect See explanation for response c.
b. Incorrect See explanation for response c.
c. CORRECT Of the specific CNS drugs identified in the survey, antipsychotics and antimanics were the only ones that were provided or prescribed more often by practitioners in medical specialty offices (e.g., psychiatrists and neurologists) than by primary care physicians.
d. Incorrect See explanation for response c.

The correct answer is: antipsychotic and antimanic drugs

205
Q

A patient whose corpus callosum has been severed as a treatment for severe epilepsy will be able to do which of the following?
Select one:

A.
verbally identify an odor that has been presented only to her right nostril

B.
identify a pen with her left hand after an image of a pen has been projected to her left visual field

C.
say the word “pen” after an image of a pen has been projected to her left visual field

D.
repeat four words that have been whispered into her left ear

A

If you encounter a question like this one on the exam, keep in mind that (1) language is controlled by the left hemisphere in most people and (2) most functions are controlled contralaterally but that olfaction is an exception to this general rule.

a. Incorrect As noted above, olfactory signals do not cross to the other side of the brain. Consequently, an odor entering the right nostril only will be sent to the right hemisphere.
b. CORRECT An image presented to the left visual field will be processed by the right hemisphere which controls the left hand. Therefore, the patient will be able to perform this task.
c. Incorrect Because information projected to the left visual field will be sent to the right hemisphere, the patient will not be able to say what she has seen.
d. Incorrect Information entering the left ear is sent to the right hemisphere. Consequently, the patient will not be able to repeat that information.

The correct answer is: identify a pen with her left hand after an image of a pen has been projected to her left visual field

206
Q

Generally speaking, compared to first-generation antipsychotics, second-generation antipsychotics (e.g., clozapine, risperidone) are:
Select one:

A.
just as likely to cause tardive dyskinesia but less likely to cause agranulocytosis or other blood dyscrasia.

B.
just as likely to cause tardive dyskinesia and more likely to cause agranulocytosis or other blood dyscrasia.

C.
more likely to cause tardive dyskinesia.

D.
less likely to cause tardive dyskinesia.

A

For the exam, you want to be familiar with the side effects of the first- and second-generation antipsychotics (which are also known as traditional and atypical antipsychotics, respectively). This information is provided in the Physiological Psychology and Psychopharmacology chapter of the written study materials.

a. Incorrect See explanation for responses b and d.
b. Incorrect Although the second-generation drugs are more likely to produce agranulocytosis and other blood dyscrasias, they are less likely to produce tardive dyskinesia.
c. Incorrect See explanation for response d.
d. CORRECT Second-generation antipsychotics are less likely than first-generation antipsychotics to produce tardive dyskinesia. (Note, however, that the risk for tardive dyskinesia is slightly higher for risperidone than for clozapine but that that the risk with risperidone is still lower than the risk with the first-generation drugs.)

The correct answer is: less likely to cause tardive dyskinesia.

207
Q

An older adult in the middle (second) stage of Alzheimer’s disease is most likely to exhibit which of the following?
Select one:

A.
apathy, incontinence, and an inability to recognize close relatives and friends

B.
restlessness and agitation, anterograde and retrograde amnesia, and flat or labile mood

C.
impaired attention and judgment, anterograde amnesia, and indifference or sadness

D.
poor judgment and insight, changes in speech and writing, and tremor

A

Answer B is correct: Alzheimer’s disease is often described in terms of three stages (see the Abnormal Psychology chapter of the written study materials). The symptoms listed in this answer are characteristic of the second stage.

Answer A: These symptoms are characteristic of the third (late) stage of the disorder.

Answer C: These symptoms are characteristic of the first (early) stage of Alzheimer’s disease.

Answer D: These symptoms are characteristic of Huntington’s disease.

The correct answer is: restlessness and agitation, anterograde and retrograde amnesia, and flat or labile mood

208
Q

Bill B., a 26-year old small business owner, has trouble completing tasks at home and work, frequently “blows his fuse” at family members and employees, and has his wife do all the paperwork at the office because he doesn’t have the patience for it. His wife says Bill never listens to her and often does things without thinking about the consequences. When asked about his past, Bill says that he often got in trouble while he was in school. He frequently got into fights and, in high school, was arrested several times for drugs and drunk driving. All through school, his teachers said he wasn’t living up to his potential. Although Bill no longer uses illegal drugs, he often drinks heavily on weekends. Based on these symptoms, the most likely diagnosis for Bill is:
Select one:

A.
Antisocial Personality Disorder.

B.
Adult Antisocial Behavior.

C.
ADHD.

D.
Bipolar I Disorder.

A

Answer C is correct: Bill’s childhood history of problem behaviors and his current attention difficulties and impulsivity are most suggestive of ADHD.

Answer A: For a diagnosis of Antisocial Personality Disorder, there should be a current pattern of antisocial behavior.

Answer B: Adult Antisocial Behavior is included in the DSM with Other Conditions That May Be a Focus of Clinical Attention and applies when the individual exhibits antisocial behavior that is not meet the diagnostic criteria for Antisocial Personality Disorder or other mental disorder.

Answer D: The continuous and long-term nature of Bill’s symptoms are more suggestive of ADHD than Bipolar I Disorder.

The correct answer is: ADHD.

209
Q

A factor analysis indicates that Test A has a factor loading of .80 for Factor 1 and a factor loading of .10 for Factor 2. Assuming an orthogonal rotation, the communality for Test A is:
Select one:

A.
1.0.

B.
.90.

C.
.70.

D.
.65.

A

When factors are orthogonal (uncorrelated), the factor loadings can be squared and summed to calculate the communality.

a. Incorrect See explanation for response d.
b. Incorrect See explanation for response d.
c. Incorrect See explanation for response d.
d. CORRECT .80 squared plus .10 squared equals .64 plus .01, or .65. The communality for Test A is .65, which means that 65% of variability in Test A scores is explained by Factors 1 and 2.

The correct answer is: .65.

210
Q

Constructional apraxia is caused by damage to the:
Select one:

A.
corpus callosum.

B.
parietal lobe.

C.
basal ganglia.

D.
temporal lobe.

A

Constructional apraxia is characterized by an inability to copy or draw figures or to arrange blocks in a pattern.

a. Incorrect See explanation for response b.
b. CORRECT Apraxia involves an inability to perform skilled movements that are not due to muscle weakness, sensory loss, general intellectual deterioration, or lack of cooperation. Most forms of apraxia (including constructional apraxia) are caused by damage to the parietal lobe.
c. Incorrect See explanation for response b.
d. Incorrect See explanation for response b.

The correct answer is: parietal lobe.

211
Q

Which of the following is the LEAST common side effect of the beta-blocker propranolol (Inderal)?
Select one:

A.
bradycardia

B.
fatigue and malaise

C.
sexual dysfunction

D.
coldness in extremities

A

The beta-blockers are associated with a number of undesirable side effects.

a. Incorrect Bradycardia occurs in about 10% of cases.
b. Incorrect Fatigue and malaise also occur in about 10% of cases and may lead to a misdiagnosis of depression.
c. Incorrect Decreased sexual ability also occurs in about 10% of cases.
d. CORRECT Coldness in the extremities occurs in less than 1% of cases. See, e.g., M. A. Fuller & M. Sajatovic (1999), Drug information handbook for psychiatry, Hudson, OH, Lexi-Comp, Inc.

The correct answer is: coldness in extremities

212
Q

Research suggests that, in general, the most effective treatment for Obsessive-Compulsive Disorder (OCD) is which of the following?
Select one:

A.
exposure with response prevention

B.
overcorrection

C.
desensitization and behavioral rehearsal

D.
desensitization with reinforced practice

A

Answer A is correct: Studies investigating the effects of various behavioral therapies have consistently found exposure with response (ritual) prevention to be the most effective treatment for OCD.

Answers B, C, and D: See explanation for answer A.

The correct answer is: exposure with response prevention

213
Q

Studies examining the therapist preferences of individuals belonging to culturally diverse groups indicates that they typically:
Select one:

A.
rank race/ethnic similarity as more important than attitude or worldview similarity.

B.
rank attitude and worldview similarity as more important than race/ethnic similarity.

C.
rank personality similarity as more important than either race/ethnic similarity or attitude similarity.

D.
rank age and gender similarity as more important than either race-ethic similarity or attitude similarity.

A

Although members of culturally diverse groups express a stronger preference for therapists who are similar in terms of race or ethnicity, race and ethnicity are not always their first consideration.

a. Incorrect This is the opposite of what some research has found.
b. CORRECT This was the finding of several studies which found that, when members of culturally diverse groups were given a choice between race/ethnic similarity and attitude similarity, most ranked attitude similarity as more important. See, e.g., D. R. Atkinson and C. E. Thompson, Racial, ethnic, and cultural variables in counseling, in S. D. Brown and R. W. Lent (Eds.), Handbook of counseling psychology, New York, John Wiley & Sons, 1992).
c. Incorrect Personality is ranked as less important than attitude or race/ethnicity.
d. Incorrect See explanation above.

The correct answer is: rank attitude and worldview similarity as more important than race/ethnic similarity.

214
Q

Which of the following individuals is most likely to receive a diagnosis of Schizotypal Personality Disorder?
Select one:

A.
a 26-year-old man who says he’s unhappy about having no close friends but doesn’t attempt to make friends with others because he believes he has telepathy and hears other people’s thoughts

B.
a 28-year old woman who says she would like to have friends and makes attempts to meet new people but has trouble maintaining relationships because of her need to maintain control of all aspects of her life

C.
a 31-year old man who expresses a lack of interest in relationships and prefers to be alone because he doesn’t derive any pleasure from friendships or social groups

D.
a 35-year-old woman who expresses unhappiness about her lack of friends but avoids them because of her fear of rejection or abandonment.

A

Answer A is correct: Schizotypal Personality Disorder is characterized by a pattern of social and interpersonal deficits involving acute discomfort with and reduced capacity for close relationships and eccentricities in cognition, perception and behavior. Individuals with Schizotypal Personality Disorder may express unhappiness with their lack of relationships but don’t pursue friendships because of their discomfort around other people and belief that they are different from others.

Answers B, C, and D: See explanation for answer A.

The correct answer is: a 26-year-old man who says he’s unhappy about having no close friends but doesn’t attempt to make friends with others because he believes he has telepathy and hears other people’s thoughts

215
Q

Which of the following is LEAST characteristic of Huntington’s disease?
Select one:

A.
anxiety, depression, and apathy

B.
a “dance-like” gait

C.
slow writhing movements

D.
apraxia, aphasia, and agnosia

A

Huntington’s disease produces a form of subcortical dementia and, therefore, has symptoms that differ somewhat from dementia due to Alzheimer’s disease and other forms of cortical dementia.

a. Incorrect Emotional and personality changes are often the first signs of Huntington’s disease.
b. Incorrect A dance-like gait is characteristic of Huntington’s disease.
c. Incorrect Slow writing movements (athetosis) are characteristic of this disorder.
d. CORRECT These are symptoms of Alzheimer’s disease and other cortical dementias.

The correct answer is: apraxia, aphasia, and agnosia

216
Q

When working with clients with severe depression, Aaron Beck is most likely to begin therapy with which of the following?
Select one:

A.
hypnosis

B.
pharmacotherapy

C.
cognitive restructuring exercises

D.
behavioral tasks

A

To answer this question, you need to be aware that Beck’s approach incorporates cognitive and behavioral techniques. You also need to note that the question is asking about patients with severe depression.

a. Incorrect See explanation for response d.
b. Incorrect See explanation for response d.
c. Incorrect See explanation for response d.
d. CORRECT Beck notes that therapy may begin somewhat differently for individuals with severe versus mild to moderate forms of depression. For clients with severe depression, it may be necessary to start with behavioral techniques that are designed to increase the client’s overall activity level (e.g., graded task assignments). For clients with mild to moderate depression, cognitive techniques are introduced during initial sessions. See, e.g., A. T. Beck and M. Weishaar, Cognitive therapy, in A. Freeman, et al., Comprehensive Handbook of Cognitive Therapy, New York, Plenum, 1989.

The correct answer is: behavioral tasks

217
Q

Responsibility for the validity of information provided by a professional scoring and interpretation service lies with:
Select one:

A.
the psychologist who uses the service.

B.
the scoring and interpretation service.

C.
the test author(s).

D.
the organization that accredited the service.

A

The use of computer-assisted scoring and interpretation services is addressed in Standard 9.09(c) of the APA’s Ethics Code.

a. CORRECT This Standard states: “(b) Psychologists select scoring and interpretation services (including automated services) on the basis of the validity of the program and procedures as well as on other appropriate considerations…. (c) Psychologists retain responsibility for the appropriate application, interpretation, and use of assessment instruments, whether they score and interpret such tests themselves or use automated or other services.”
b. Incorrect See explanation for response a.
c. Incorrect See explanation for response a.
d. Incorrect See explanation for response a.

The correct answer is: the psychologist who uses the service.

218
Q

Absence (petit mal) seizures appear to have their origin in which area of the brain?
Select one:

A.
motor cortex

B.
occipital lobe

C.
amygdala

D.
thalamus

A

Absence seizures are characterized by an altered level of consciousness without significant motor symptoms.

a. Incorrect See explanation for response d.
b. Incorrect See explanation for response d.
c. Incorrect See explanation for response d.
d. CORRECT Research with animals and humans indicates that petit mal seizures begin in the thalamus, which is considered the “gateway to consciousness.” For example, research with rats has linked an overabundance of GABA receptors in the thalamus to petit mal seizures.

The correct answer is: thalamus

219
Q

Research has found that our self-image remains relatively stable due, in part, to the fact that we seek, elicit, and remember information that confirms our beliefs about ourselves. This finding is predicted by which of the following?
Select one:

A.
constancy bias

B.
self-serving bias

C.
false consensus bias

D.
confirmation bias

A

The question is referring to the tendency to search for and recall information that confirms our preconceptions.

a. Incorrect This is a distractor (made-up term).
b. Incorrect The self-serving bias is the tendency to take credit for our successes but blame external factors for our failures.
b. Incorrect The false consensus bias is the tendency to overestimate the degree to which the beliefs of others are similar to our own.
d. CORRECT This tendency is referred to as the confirmation (confirmatory) bias.

The correct answer is: confirmation bias

220
Q

Sweating, speeded up mental processes, inability to sleep or relax, trembling, nervousness, palpitations, and loss of weight despite increased food intake are characteristic of which of the following endocrine disorders?
Select one:

A.
hyperglycemia

B.
hyperthyroidism

C.
adrenal insufficiency

D.
hypoparathyroidism

A

For the licensing exam, you want to be familiar with the symptoms of the endocrine disorders listed in the answers to this question.

a. Incorrect Hyperglycemia is due to excessive glucose in the blood. Its symptoms include fatigue, polydipsia, polyuria, polyphagia, weight loss, poor wound healing, and recurrent infections.
b. CORRECT The symptoms listed in this question are characteristic of hyperthyroidism, which is caused by excessive secretion of thyroxin by the thyroid gland.
c. Incorrect Adrenal insufficiency (also known as Addison’s disease) is due to a lower-than-normal production of cortisol by the adrenal glands and is characterized by weight loss, orthostatic hypotension, weakness, fatigue, and hyperpigmentation.
d. Incorrect Hypoparathyroidism is caused by lower-than-normal levels of parathyroid hormone. Its symptoms include muscle cramping and twitching, tingling in the lips and fingers, hair loss, and dry skin.

The correct answer is: hyperthyroidism

221
Q

Joining and accommodating are the initial steps in __________ family therapy.
Select one:

A.
object relations

B.
strategic

C.
Milan systemic

D.
structural

A

For the exam, you want to have joining and accommodating associated with Minuchin’s structural family therapy.

a. Incorrect See explanation for response d.
b. Incorrect See explanation for response d.
c. Incorrect See explanation for response d.
d. CORRECT Minuchin viewed joining and accommodating as necessary initial processes that reduce family members’ resistance to change.

The correct answer is: structural

222
Q

The basic assumptions and procedures of motivational interviewing (Miller & Rollnick, 1991) were derived from:
Select one:

A.
Sullivan’s interpersonal therapy and Bandura’s notion of self-efficacy.

B.
Sullivan’s interpersonal therapy and Meyer’s psychobiological approach.

C.
Rogers’ person-centered therapy and Maslow’s need hierarchy.

D.
Rogers’ person-centered therapy and Bandura’s notion of self-efficacy.

A

As its name implies, motivational interviewing was developed for clients who are ambivalent about changing their behavior.

a. Incorrect See explanation for response d.
b. Incorrect See explanation for response d.
c. Incorrect See explanation for resposne d.
d. CORRECT As noted in the Clinical Psychology chapter of the written study materials, motivational interviewing stresses therapist empathy, reflective listening, and responding to a client’s resistance in a nonconfrontational way and addresses the client’s beliefs about his/her ability to change.

The correct answer is: Rogers’ person-centered therapy and Bandura’s notion of self-efficacy.

223
Q

Moffitt (1993) attributes the life-course-persistent type of antisocial behavior to which of the following?
Select one:

A.
a maturity gap

B.
behavioral disinhibition

C.
neurological vulnerability

D.
a personality disorder

A

Answer C is correct: Moffitt (1993) distinguishes between two types of antisocial behavior that differ in terms of onset, symptom severity, and etiology. She attributes the adolescence-limited type to a maturity gap, and the life-course persistent type to a combination of neurological vulnerability, a difficult temperament, and a criminogenic environment (e.g., parental criminality, inept parenting).

Answers A, B, and D: See explanation for answer C.

The correct answer is: neurological vulnerability

224
Q

In his study on the effects of therapy fees on satisfaction with therapy, a social psychologist finds that clients who have to work overtime or get a second job to pay for therapy generally express greater satisfaction than wealthy clients, clients whose insurance covers most of their fee, or clients who are receiving therapy for free or at low cost. This finding provides support for which of the following?
Select one:

A.
the overjustification hypothesis

B.
cognitive dissonance theory

C.
the approach-avoidance conflict

D.
equity theory

A

The results of the study indicate that people who “struggle” for therapy say it gives them greater satisfaction.

a. Incorrect The overjustification hypothesis is used to explain why intrinsic motivation decreases after external rewards have been applied.
b. CORRECT This finding is consistent with other research on cognitive dissonance, including studies showing that students say they like a dull club more when they have experienced a difficult initiation.
c. Incorrect The approach-avoidance conflict doesn’t really fit the situation described in this question.
d. Incorrect Equity theory makes predictions about how people perform in situations they perceive to be either equitable or inequitable.

The correct answer is: cognitive dissonance theory

225
Q

The Final Conclusions of the American Psychological Association Working Group on Investigation of Memories of Childhood Abuse (APA, 2000) recommends that, when a therapy client seeks hypnosis as a means of retrieving forgotten memories of childhood abuse, the therapist should:
Select one:

A.
provide such services only if he/she has adequate training and experience.

B.
provide such services only if he/she has adequate training and experience and seeks supervision in order to ensure objectivity.

C.
advise the client of the potential risks of hypnosis in this context before using it.

D.
advise the client that hypnosis is not an appropriate procedure for retrieving forgotten memories.

A

The Working Group’s statement recognizes that memories of childhood abuse can be forgotten and later recalled and provides guidelines for ensuring that recalled memories are not false.

d. CORRECT This response is very similar to the actual language of the Final Conclusions, which advises therapists to warn clients that hypnosis increases the risk for pseudo memories as well as confidence in the veracity of those memories. (Note that the conclusion reached in this document is similar to conclusions reached elsewhere. See the Clinical Psychology chapter of the written study materials for additional information on this issue.)

The correct answer is: advise the client that hypnosis is not an appropriate procedure for retrieving forgotten memories.